[go: up one dir, main page]

100% found this document useful (1 vote)
450 views58 pages

Gynae & Obs Bcqs Booklet

1. A 24-year-old woman who is requesting contraception 6 weeks postpartum would be best advised to use oral contraceptives, as these are not associated with an increase in dysmenorrhea unlike copper IUDs. 2. Vitamin D should be increased in the diet of a girl with premenstrual syndrome. 3. Vaginismus is defined as pain during intercourse due to vaginal tightness and pain during penetration.

Uploaded by

Kiran Ayub
Copyright
© © All Rights Reserved
We take content rights seriously. If you suspect this is your content, claim it here.
Available Formats
Download as PDF, TXT or read online on Scribd
100% found this document useful (1 vote)
450 views58 pages

Gynae & Obs Bcqs Booklet

1. A 24-year-old woman who is requesting contraception 6 weeks postpartum would be best advised to use oral contraceptives, as these are not associated with an increase in dysmenorrhea unlike copper IUDs. 2. Vitamin D should be increased in the diet of a girl with premenstrual syndrome. 3. Vaginismus is defined as pain during intercourse due to vaginal tightness and pain during penetration.

Uploaded by

Kiran Ayub
Copyright
© © All Rights Reserved
We take content rights seriously. If you suspect this is your content, claim it here.
Available Formats
Download as PDF, TXT or read online on Scribd
You are on page 1/ 58

1. A 24-years old G2P2 is requesting 6.

Ovarian neoplasms most commonly arise from


contraception 06 weeks postpartum. Her which of the following cell lines:
history is unremarkable except for significant A. Ovarian epithelium T
primary dysmenorrhea. Which of the following B. Metastatic disease
contraception methods is most closely C. Ovarian sex cords
associated with an increase in dysmenorrhea: D. Ovarian germ cells
A. Copper containing IUCD T E. Ovarian stroma
B. Cervical cap
C. Oral contraceptives (OCs) 7. The average age of menopause is:
D. Male condoms A. 40
E. Progestin only pill (the minipill) B. 45
C. 50 T
2. Which of the following vitamin you will D. 55
increase in the diet of a girl having E. 60
premenstrual syndrome:
A. Vitamin C 8. If germ cells fail to enter the developing genital
B. Vitamin D T ridge, which of the following may occur:
C. Vitamin A A. Ovarian choriocarcinoma
D. Vitamin E B. Ovarian teratoma
E. Vitamin B C. Testicular feminization
D. Gonadal agenesis / streak gonads T
3. The definition of vaginismus is: E. Ectopic pregnancy
A. Intact hymenal membrane
B. Pain during menses 9. A 17-years old girl is brought to the OPD due to
C. Absence of vagina failure of menstruation. She is 5 feet and 7
D. Pain during intercourse inches tall. Initial examination shows breast in
E. Vaginal tightness & pain during tanner stage-I development scanty pubic hair,
intercourse T no axillary hair, local examination reveals blind
ended vagina karyotype repant is 46XY. What
4. Which of the following is a cornerstone for is the most probable diagnosis:
detection of ovarian neoplasia: A. Androgen insensitivity syndrome T
A. Pelvic examination B. XY engynetic failure
B. CA-125 T C. Gonadal agenesis
C. Pelvic ultrasound D. Mayer Rokitansky Kurter Hausen syndrome
D. Human chorionic gonadotropin (hCG) E. Turner syndrome
E. Alpha fetoprotein
10. A 28-years old woman P1+2 presented with
5. A 25-years old lady has come with positive H/O amenorrhea 1½ month followed by mild
pregnancy test but despite of three ultrasounds bleeding P/V. On ultrasound no gestational sac
one week apart failed to show pregnancy in seen, ovaries were enlarged with multiple cysts.
uterus or in adnexa. What is the name of the Her β-HCG was raised. The findings are
condition: consistent with:
A. Threatened miscarriage A. Theca leutin cysts T
B. Pregnancy of unknown location B. Polycystic ovaries
C. Chemical pregnancy T C. Corpus leuteal cyst
D. Blighted ovum D. Follicular cysts
E. Missed miscarriage E. Endometriotic cyst

Page | 1
11. A 27-years old P0A0 underwent myomectomy C. Paramesonephric duct
due to fibroid uterus. As a routine on which D. Wolffian duct
postoperative day her hemoglobin should be E. Primordial follicles
repeated:
A. 1st day 17. A 60-years old P4A0 has been offered
B. 2nd day T abdominal hysterectomy due to degenerating
C. 3rd day fibroid. Which of the following abdominal
D. 4th day hysterectomy will be best for her:
E. 5th day A. Subtotal hysterectomy
B. Modified radical hysterectomy
12. A 45-years old woman has come to gynecology C. Radical hysterectomy
clinic saying that she is having night sweats. D. Total hysterectomy
What is the reason her daughter wants to know E. Total hysterectomy + bilateral salpingo-
as she is studying medicine. You explain: oophorectomy T
A. Lack of ovulation causing progesterone
deficiency 18. A patient currently on COCs is being scheduled
B. Its normal sequel of life, all women have it to undergo a laparoscopic myomectomy.
C. Lack of balanced diet affecting Regarding the perioperative management of
thermoregulatory center her COCs, you should counsel her to do which
D. Lack of exercise leading to endorphin of the following:
deficiency A. Continue her contraceptives use until after the
E. Lack of oesrogenic modulation on serotine procedure
receptor T B. Discontinue her contraceptives 14 days prior
to surgery
13. A young woman presented in early pregnancy C. Discontinue her contraceptives at least 30
clinic with positive pregnancy test and excessive days prior to surgery T
vomiting. Her ultrasound reveals no definite D. Discontinue her contraceptives 07 days prior
fetus however uterus was filled with cystic to surgery
spaces. What is your diagnosis: E. Discontinue her contraceptives the day of the
A. Missed miscarriage surgery
B. Biochemical pregnancy
C. Ectopic pregnancy 19. A 34-years old woman para 1+0 presented with
D. Molar pregnancy T heavy menstrual bleeding for last six months.
E. Blighted ovum P/A examination reveals 20 weeks size mass
arising from hypogastrium. Most likely
14. Heavy menstrual bleeding after the age of 45 diagnosis is:
years is cause of concern because of risk of: A. Endometriosis
A. Endometrial polyps B. Ovarian malignancy
B. Endometrial hyperplasia C. Uterine fibroid T
C. Carcinoma of endometrium D. Mesenteric cyst
D. Leiomyosarcoma of uterus E. Pelvic inflammatory disease
E. Adenomyosis T
20. A 35-years old woman presented at gynecology
15. A 38-years old P4A0 is on oral contraceptive OPD with C/O irregular menstrual bleeding.
pills for last two years. She has been offered by Her TVS revealed endometrial polyp. Best
her consultant abdominal hysterectomy due to management option is:
multiple fibroid and heavy menstrual bleeding. A. Hysteroscopic guided polypectomy T
How many weeks prior surgery she should stop B. HRT
oral contraceptive pills: C. Follow up after six months
A. One week D. Dilatation & curettage
B. Two weeks E. TAH & BSO
C. Three weeks
D. Four weeks 21. Vulvo-vaginal thrush is common in women of:
E. Five weeks T A. Post menopause
B. Any age group
16. The precursor of female genital tract is: C. Adolescent period
A. Mesonephric ridge D. Peri menopause
B. Mullerian duct T E. Reproductive age T

Page | 2
22. A primigravida with miscarriage asks you E. Heavy menstrual bleeding (HMB)
when a surgical management is indicated for
miscarriage. The most appropriate reply would 28. An obese woman with history of DVT in family
be: come to you for contraceptive advice. Which
A. Cervical os has opened one of the following you will never advise:
B. Whenever woman wants A. Depoprovera
C. Persistent excessive bleeding T B. Combined hormonal continuous (CHC)
D. As soon as diagnosis is made patch T
E. Cervical os is tightly closed C. LNG-IUS
D. POP
23. A 36-years old woman P4+0 presents with E. Progesterone
hegnler heavy periods for the last 08 months.
Her examination and ultrasound reveals 29. Oxytocin infusion prior to completion of
normal size uterus, non-tender, in addition she suction evacuation of molar pregnancy is not
wants contraception. What is the best treatment recommended due to the risk of:
option for her: A. Severe maternal hypotension
A. OCPs B. Tissue embolization T
B. OCP for 03 months C. Excessive hemorrhage
C. LNG-IUS T D. Retention of molar tissue
D. Progesterone only pills E. Prolong operative time
E. Endometrial ablation
30. MEC wheel is system developed by WHO to
24. A 45-years old woman para 05 has irregular prescribe:
vaginal bleeding for few months. Her hb: is 7.3 A. Long acting progesterone
gm. In the following which investigation is most B. Estrogen patch
appropriate to diagnose her problem: C. IUCD
A. MRI D. Contraception T
B. Diagnostic curettage/ endometrial E. Implants
sampling T
C. Ultrasound exam of pelvis 31. Commonest etiological factor for early
D. CT scan miscarriage is likely to be:
E. Blood complete picture A. Infections
B. Chemicals & drugs
25. A 53-years old lady with postmenopausal C. Chromosomal abnormalities T
bleeding diagnosed as case of endometrial D. Medical disorders
cancer. Her MRI shows stage-Ia. The E. Uterine abnormalities
recommended treatment for this patient is:
A. TAH & BSO T 32. A copper IUD inside the uterine cavity prevent
B. Radiotherapy a woman becoming pregnant by:
C. Chemotherapy A. Spermicidal effect
D. Radical hysterectomy B. Preventing fertilization
E. Wertheim hysterectomy C. Causing an ovulation
D. Making cervical mucus thick
26. In a woman with epilepsy the efficacy of E. Inflammatory reaction T
hormonal contraceptives will decrease if she is
taking: 33. A 30-years old woman para 0+0 attended
A. Antipsychotic drugs fertility clinic. Pelvic ultrasound shows 18
B. Electroconvulsive therapy weeks size intramural fibroid. The best
C. Carbamazepine T treatment option is:
D. Diazepam A. Uterine artery embolization
E. Folic acid B. GnRH agonist
C. Progesterone pills
27. What is the term used for a 40 years old woman D. Hysterectomy
having regular but heavy menstrual loss: E. Myomectomy after GnRH agonist T
A. Metrorrhagia
B. Dysfunctional uterine bleeding
C. Perimenopausal bleeding
D. Menorrhagia T
Page | 3
34. A 62-years old woman attended clinic with 39. Initial management of inversion of uterus is:
large prolapse, she had abdominal A. Hydrostatic method of Osulivan’s
hysterectomy at age of 42 due to fibroids, again B. Hysterectomy
at age of 54 she had laparotomy due to large C. Manual replacement of uterus under
ovarian cyst and that surgery was difficult due anesthesia T
to dense abdominal adhesions. On examination D. Halton’s procedure
there is large vault prolapse which extended E. Laparotomy
beyond the introitus. Which of the following is
best surgical option for her: 40. Among the given clinical presentation of PCOS.
A. Abdominal sacrocolpopexy The most common is:
B. Vaginal repair with sacrospinous fixation A. Hirsutism
C. Anteroposterior repair B. Asymptomatic
D. Colpocleisis C. Acanthosis nigricans
E. Insertion of vaginal pessary D. Obesity
E. Subfertility T
35. A 30-years old multiparous woman request you
to give her medicine for dysmenorrhea till she 41. When sampling the cervix for a pap smear it is
finds a consultant. Your best advise will be: critical to sample which area since it is the most
A. Mefenamic acid T likely source of cervical cancer. Where do most
B. Medroxy-progesterone acetate cervical cancers arise:
C. Low fat diet A. In the endocervix
D. Desogestrol B. At the external os
E. COCP C. On the portio-vaginalis
D. At the squamo-columnar junction T
36. You diagnosed a 32-years old woman with E. At the internal os
HMB. Which of the test you should not
perform: 42. A young lady has been kept for evacuation due
A. Coagulation profile to molar pregnancy. To reduce the chances of
B. Full blood count (FBC) perforation of uterus the best approach is:
C. Endometrial biopsy T A. Suction evacuation T
D. Hormone tests B. Manual vacuum aspiration
E. High vaginal swab C. Cervical priming with misoprostol
D. Ultrasound guided suction evacuation
37. A 28-years old woman had suction evacuation E. Dilatation & curettage
for molar pregnancy. Histopathology showed
partial molar pregnancy. She was advised for 43. A girl exhibiting amenorrhea along with other
regular follow up. In the following which features of PCO because of:
investigation is most significant in her following A. Elevated insulin resistance
program: B. Chronic anovulation
A. Blood CP C. Androgens T
B. β-HCG T D. Weight gain
C. TSH E. Increased LH
D. RFT
E. Chest X-ray 44. Which of the following factors is protective
against endometrial hyperplasia:
38. The patient has biopsy proven CIN-III. She A. Obesity
requests cryotherapy for treatment. B. Unopposed exogenous estrogen therapy
Cryotherapy is appropriate to consider in C. Early menarche or late menopause
which clinical circumstance: D. Oral contraceptive pills (OCPs) T
A. A patient who wishes to preserve fertility E. Tamoxifen
B. CIN-III
C. A patient with well-circumscribed, small 45. Most common vulval cancer is:
lesion of mild dysplasia (CIN-I) T A. Malignant melanoma
D. Invasive carcinoma B. Clear cell carcinoma
E. An HIV positive patient C. Basal cell carcinoma
D. Squamous cell carcinoma T
E. Adenocarcinoma

Page | 4
46. Benign dermatological vulval conditions are 52. A woman refuses contraception by saying it is
more common during the period of: religiously prohibited to destroy a fertilized
A. Pregnancy ovum. You convince her by saying
B. Perimenopause contraceptive pills work by preventing:
C. Childhood A. Ovulation T
D. Postmenopause T B. Fertilization
E. Lactation C. Cervical secretions
D. Sperm motility
47. An office hysteroscopic endometrial biopsy is E. Menstrual cycle
indicated for HMB when:
A. Uterus is retroverted 53. A consultant has ordered termination of
B. Transvaginal ultrasound is not available pregnancy in a nulliparous woman because of
C. Endometrial biopsy (EB) sample insufficient some medical indication. Which of the test you
for histopathology T must send before the procedure:
D. Endometrial biopsy (EB) shows hyperplasia A. HSV for chlamydia
E. Transvaginal ultrasound show large fibroid B. HIV testing
C. Blood grouping with Rh factor T
48. Emergency contraception can be effective if D. CBC
administered up until how long after E. VDRL for STIs
intercourse:
A. 24 hours 54. A 34-years old woman with amenorrhea of 06
B. 48 hours weeks has pain in abdomen and slight vaginal
C. 72 hours bleeding. On examination cervical os is tightly
D. 04 days closed but she felt severe pain on examination.
E. 05 days T What is the most likely diagnosis:
A. Threatened miscarriage T
49. Which of the following advantage multifilament B. Missed miscarriage
suture has over monofilament: C. Cervicitis
A. Knot is more secure D. Ectopic pregnancy
B. Cause less tissue reaction E. Pelvic inflammatory disease
C. Tensile strength is more T
D. Is non absorbable 55. A 32-years old para 04 taking a COCPs for
E. Decrease risk of infection contraception but forgets at times, is having
irregular bleeding for last 15 days. Her LMP
50. In patient with uterovaginal prolapse, was six weeks ago. Which of the following lab
rectovaginal examination is important: test is most appropriate:
A. When woman refuse vaginal examination A. TSH
B. When woman with pelvic mass B. Free T4
C. To differentiate between rectocele & C. B-HCH T
enterocele T D. CBC
D. When ultrasound is not available E. Prolactin
E. For all women with incontinence
56. A 35-years old para 03 presents in STI clinic
51. An obese girl with PCO having acne and with history of vaginal discharge for last 02 to
hirsutism wants to avoid conception till she 03 years. On per speculum examination cervix
loses half of her weight, for at least one year. is inflamed and smeared with mucopurulent
What will be best method for her: vaginal discharge. Subsequent vulvovaginal
A. Intrauterine device swabs and NAAT testing confirms chlamydia
B. Combined oral contraceptive pill T trachomatis infection. Following statement is
C. Progesterone implant true about this infection:
D. Long acting progesterone injections A. Reactive arthritis is commonly seen in
E. POP females
B. It doesn’t lead to long term complications
C. Testing of sexual partner is usually not
required
D. Usually it affects lower genital tract
E. It is the most common bacterial STI T

Page | 5
57. Radical radiotherapy is given instead of D. Cholesterol lecithin-cardiolipin antigen test
surgery in vulval cancer when women are: E. RPR test
A. Stage II disease
B. With lymph node involvement 63. A 42-years old para 3+0 referred to gynecology
C. Young women oncology clinic for stage-Ib1 cervical
D. Having tumor confined to vulva malignancy. What is the standard management
E. Unfit for surgery option:
A. Local excision of disease
58. A pregnant lady at 08 weeks came in antenatal B. Internal beam radiotherapy
clinic with H/O hyperemesis gravidarum in C. Chemotherapy
previous pregnancy, now complaining of D. Radical hysterectomy & pelvic lymph node
nausea and vomiting twice a day, she is afraid dissection T
of her previous experience. What advice would E. External beam radiotherapy
you like to give her:
A. Prescribe some IV fluid as outpatient 64. Ovaries are likely to metastasize in certain gut
attendance tumors. Following is the example of metastatic
B. Reassurance that severe symptoms are ovarian tumor:
unlikely to recur A. Endometroid tumor
C. Treatment with prothiazine if symptoms B. Clear cell tumor
worsen T C. Krukenberg tumor T
D. Treatment with corticosteroid if symptoms D. Choriocarcinoma
worsen E. Teratoma
E. Admit in hospital
65. Asherman syndrome is caused by damage to
59. Misoprostol for termination of pregnancy can basal layer of endometrium. In which of the
be used in outpatient setting before: following procedure basal layer of
A. 06 weeks endometrium is deliberately destroyed:
B. 12 weeks A. Endometrial ablation T
C. 14 weeks B. Fractional curettage
D. 16 weeks C. Hysteroscopic endometrial sampling
E. 09 weeks T D. Conventional curettage
E. Manual vacuum aspiration
60. A 35-years old woman with low risk of
malignancy requires outpatient endometrial 66. Vulvodynia is a:
sampling advised due to heavy menstrual loss. A. Malignant disease
What will be the best method for the purpose: B. Normal physiological change
A. Pipple biopsy C. Painful condition of vulva T
B. Office hysteroscopy D. Skin disease
C. Taking sample on slide with cytobrush E. Infection
D. Laparoscopy T
E. Evacuation & examination under deep 67. Toxic shock syndrome has been associated with
sedation which of the following contraceptive methods:
A. OCs
61. WHO reference volume for semen analysis B. Male condoms
show a normal semen volume of: C. Progestin-only pill (the mini pill)
A. > 1.1 ml D. IUCDs
B. > 1.5 ml T E. Cervical cap T
C. > 5 ml
D. > 3 ml 68. To prevent iso-immunization in rhesus negative
E. > 2 ml women in early pregnancy. Anti-D should be
administered after:
62. A 30-years old woman presents in outpatient A. Threatened miscarriage
department with test positive for syphilis using B. Surgical evacuation of RPOCs
VDRL screening test. Which of the following is C. Spontaneous miscarriage
definitive test for syphilis: D. Pregnancy test is positive T
A. Ziehl-Nelson stain E. Incomplete miscarriage
B. PCR
C. FTA-ABS test T
Page | 6
69. A young girl of 13 years is having irregular C. Cognitive dysfunction
bleeding since menarche. The most likely cause D. Dementia
is: E. Osteoporosis T
A. PCO
B. Ovarian cyst 75. A 35-years old woman with crippling low back
C. Blood dyskaryosis T pain due to endometriosis. She should have a
D. Uterine polyp definite treatment with:
E. Hypo-thalamo-pituitary axis A. Oophorectomy with cryopreservation
B. Treatment with GnRh analogues
70. Which of the following methods can be effective C. Total abdominal hysterectomy with BSO
against HMB, acne and PMS along with D. NSAID for a pain relief T
contraception: E. Danazol or Gestrinone for 06 months
A. Implant
B. COCP T 76. Delayed puberty is defined as no secondary
C. IUD sexual characteristics by which one of the
D. Condom following age:
E. POP A. 12
B. 10
71. Which of the following reflects the etiology of C. 14
cervical dysplasia and cervical cancer: D. 18
A. There is a strong genetic component to the E. 16 T
development of cervical cancer
B. Human papillomavirus (HPV) is the major 77. In routine investigation of recurrent first
causal agent T trimester marriages, what investigation is not
C. They are associated with obesity advised as first line:
D. They are associated with nulliparity A. Anti-phospholipid antibodies
E. They are the direct result of cigarette smoking B. Karyotype of products of conception
C. Pelvic ultrasound scan
72. A 35-years old woman present with multiple D. OGTT T
small ulcerated lesions on vulva and urinary E. Parental karyotype
retention. The most likely infection would be:
A. HPV (human papilloma virus) 78. A 60-years old woman presents in the OPD
B. Candiasis with postmenopausal bleeding. What would be
C. HSV (herpes simplex virus) the immediate next step in her management:
D. Trichomonas vaginalis A. Diagnostic D & C
E. Syphilis B. Pelvic ultrasonography
C. Pap smear
73. A 36-years old woman attend the gynecology D. Transvaginal sonography T
clinic for troublesome clear vaginal discharge. E. Hysteroscopy
P/S examination revealed raw area on cervix.
However no any pathology found on cervical 79. Combined oral contraceptives should not be
cytology. STI screening is also unremarkable. given to:
The most likely diagnosis is: A. Women with breast cancer T
A. Cervical malignancy B. Women on insulin
B. Cervical stenosis C. Women with carcinoma cervix
C. Cervical ectropion T D. Women with hypertension
D. Cervical polyp E. Nulliparous women
E. Cervical fibroid
80. Ampullary region of the fallopian tube is the:
74. A 51-years old lady banker by profession A. Place where fertilization occurs T
presents in menopause clinic requesting for B. Least common site of ectopic pregnancy
HRT. She is known case of rheumatoid arthritis C. Thick walled structure
and on steroids for a long time. She smokes 8- D. Narrowest part of tube
10 cigarettes per day. On examination her BMI E. Smallest part of tube
is 20 kg/m2. All of the following factors in her
history increase her risk of:
A. Psychological disorders
B. Cardiovascular disease

Page | 7
81. Which of the following infection is most likely 86. A 20-years old girl reported in emergency with
associated with tubal disease: history of unsafe abortion 01 week ago and H/O
A. Proteus species high grade fever yesterday. On examination her
B. Chlamydia trachomatis T temperature is 90˚F and she is tender on pelvic
C. Bacteroids species examination. U/S examination reveals retained
D. Trichomonas vaginalis products within the uterus. The first step you
E. Treponema pallidum take is:
A. Arrange for evacuation 06 hours after
82. A 35-years old woman with crippling low back injectable antibiotics
pain due to endometriosis. She should have a B. Arrange for injectable antibiotics & vaginal
definite treatment with: misoprostol T
A. NSAID for a pain relief C. Call her after a 5th day course of oral
B. Total abdominal hysterectomy with BSO T antibiotics
C. Oophorectomy with cryopreservation D. Arrange for emergency evacuation
D. Danazol or gestrinone for 06 months E. Admit for injectable antibiotics
E. Treatment with GnRh analogues
87. A 32-years old G0P0 patient presents
83. A patient presents approximately 10 years complaining of secondary dysmenorrhea that is
postmenopausal with complaints of pressure increasing in severity. The pain is triggered by
vaginally and the sensation that something is deep thrusting with coitus. Which of the
falling out. When told she has a fallen uterus. following is the most common cause of deep
She wonders if it is due to the damage from her thrust dyspareunia:
round ligaments since she had a great deal of A. Vestibulitis
round ligament pain during her pregnancies. B. Atrophic change
Which of the following ligaments provide the C. Vaginismus
most support to the uterus in terms of D. Depression
preventing prolapse: E. Endometriosis T
A. Broad ligaments
B. Cardinal ligaments T 88. What should therapy for threatened abortion
C. Arcuate ligament include:
D. Round ligament A. Restricted activity
E. Utero-ovarian ligaments B. Progesterone intramuscular (IM) T
C. Dilatation & curettage (D&C)
84. A 22-years old lady recently married girl D. Prolonged bed rest
reported to her gynecologist complaining of E. Prostaglandin suppositories
dyspareunia and pelvic discomfort. On
examination vulva is smeared with colorless 89. Potential outcome of tubal implaominal
discharge whereas mucopurulent discharge pregnancy station is:
coming from cervix. Chlamydial infection can A. Abdominal pregnancy T
be diagnosed with 100% accuracy by: B. Persistence of tubal hematoma
A. Real time polymerase chain reaction C. Erosion of tube leading to rupture of tube
B. Nucleic acid amplification technique T D. Spontaneous involution
C. Combo 2s BD probetec E. Embedding at fimbrial end of tube
D. Slide test
E. Culture of organism 90. A patient para 2+0 came in OPD with complain
of irregular painless growth in perineal area.
85. What is the single most important factor in On evaluation there is irregular polypoidal
determining the female reproductive outcome: painless growth on vulval area. Which
A. Female age T organism cause this type of growth:
B. Anti-mullerian hormone levels A. HPV type 06 & 11
C. Serum estradiol levels B. Candida albican
D. Previous pregnancies C. Trichomoniasis
E. Ovarian follicular count D. Treponema pallidum
E. HPV type 16 & 18 T

Page | 8
91. A 35-years old woman requires outpatient 96. A patient in her forties presents with irregular
endometrial sampling advised due to heavy bleeding. You want to do an endometrial
menstrual loss. What will be the best method biopsy, she would prefer not to have the
for the purpose: procedure unless it is likely to show important
A. Pipple biopsy pathology. An endometrial sampling is likely to
B. Office hysteroscopy be reported as showing endometrial
C. Taking sample on slide with cytobrush hyperplasia in a patient who is which of the
D. Evacuation & examination under deep following:
sedation A. Using a copper intrauterine contraceptive
E. Laparoscopy T device (IUCD)
B. Using cyclic combination (OCs)
92. A 49-years old lady para 8+0 all SVD’s. She has C. Postmenopausal T
history of two difficult deliveries at home. She is D. Using DMPA
K/C of hypertension and diabetes since 10 E. Obese
years. She smokes 01 pack of cigarette per day.
Now she came with complain something coming 97. A young couple is interested in prenatal
out of vagina. Which of the following is risk diagnosis by NIPT DNA. You tell them it is
factor in her case: used to:
A. Multiparity A. Know the fetal gender
B. Hypertension B. Select patient for invasive tests
C. Home delivery C. Detect fetal
D. Smoking D. Amplify fetal DNA for PCR
E. Old age T E. Detect aneuploidy T

93. A 28-years old para 01 complains of painful 98. A 50-years old lady para 07 have an invasive
menses and painful intercourse. She has regular carcinoma of cervix. Which involved the upper
menses and no history of STDs. Which of the third of vagina but has not extended the
following tests would most likely identify parametria. The stage of disease will be:
etiology of infertility: A. II A T
A. Laparoscopy B. III A
B. Hysterosalpingogram C. I
C. Semen analysis T D. III B
D. Progesterone E. II B
E. Basal body temperature
99. A mother brings her 04 years old daughter in
94. A 65-years old lady presents in outpatient clinic OPD for complaints of itching. What is the
with ultrasound showing left sided ovarian cyst diagnosis:
measuring 4.8 x 4.2 cm. There is no septa or A. Physiologic leucorrhea
solid component. The other ovary and uterus B. Nonspecific
are normal. What should be your advice to her: C. Foreign body
A. Repeat ultrasound in 03 months D. Lichen sclerosis T
B. Reassurance E. Trauma
C. Repeat ultrasound in 04 weeks
D. CT scan 100. You are counceling a couple about factors that
E. Laparoscopic removal T can affect fertility. Which of the following
factors adversely affect spermatogenesis:
95. A 30-years old lady P2 lastborn 02 years ago A. Boxer shorts T
presents in gynecology clinic with history of B. Swimming
heavy regular menstrual bleeding for last 03 C. Febrile illness
months. Her pelvic examination is D. Exposure to cold
unremarkable and USG of pelvis is normal. E. Weekly intercourse
What will be the first line of treatment in this
case: 101. Vaginal PH is decreased:
A. Cyclical progesterone A. Women taking oral contraceptive pills T
B. Danazol B. During menstruation
C. Oral contraceptive pills C. At menopause
D. Ethamsylate D. After intercourse
E. Tranexemic acid T E. Due to cervical secretions

Page | 9
102. A patient being treated for prothrombin E. Presence of ulcer on the surface of
deficiency develops abnormal uterine bleeding. prolapse T
An anatomic lesion has been ruled out. Further
management to control the bleeding should 108. A 20-years old para 0+1 came with complain of
begin with which of the following: 10 weeks gestational amenorrhea and bleeding
A. OC pills P/V with passage of clots since 02 hours. O/E
B. Gonadotropin releasing hormone (GnRH) BP is 90/60 mmHg, pulse is 100 beats/min,
antagonists T cervical os is open, RPOCs felt. What is the
C. Medroxyprogesterone acetate appropriate treatment of this case:
D. Transdermal estradiol A. Do U/S pelvis
E. Conjugated equine estrogens B. Give misoprostol tablet T
C. Perform MVA
103. Which of the following is a cornerstone for D. Do evacuation
detection of ovarian neoplasia: E. Start syntocinon infusion
A. Human chorionic gonadotropin (hCG)
B. CA-125 T 109. Normal ovaries are best imagined by:
C. Pelvic ultrasound A. Computerized tomographic scan
D. Pelvic examination B. Trans-vaginal ultrasound T
E. Alpha fetoprotein C. Trans-abdominal ultrasound examination
D. X-ray KUB
104. A 35-years old para 08 last born 03 years using E. Doppler ultrasound
copper-T IUD for contraception is complaining
of heavy menstrual loss at regular intervals. 110. A 47-years old woman complains of postcoital
Her BP is 130/90, she looks pale, her Hb is 08 bleeding, nearly as heavy as menses. Which of
gm/dl. She is satisfied with IUD but wants to the following is the most likely origin of her
switch to another similarly convenient and bleeding:
effective method. What would you suggest her: A. Cervical infection
A. LNG intrauterine system T B. Cervical ectropion
B. Combined oral contraceptive pill C. Cervical carcinoma
C. Depot progesterone D. Cervical nabothian cysts
D. Multi-load IUD E. Cervical polyps T
E. Cyclical progesterone
111. A 35-years old woman attends the gynecology
105. Hypothalamic pituitary dysfunction is best clinic. She was advised for cervical screening.
characterized by: She wants to know the best method for cervical
A. Normal ranges of gonadotropin & estrogen cancer should be:
B. Low estrogen and normal prolactin A. Conventional pap smear T
C. Low basal gonadotropin T B. Cytology with HPV
D. Low progesterone and estrogen C. Colposcopy
E. High gonadotropin and low estrogen D. Liquid based cytology
E. HPV testing
106. Uterine leiomyomata are thought to arise from
which of the following: 112. A primigravida at 11 weeks of gestation is
A. Degenerative uterine smooth muscle admitted in emergency with heavy vaginal
cells T bleeding associated with pain. On pelvic
B. Vascular smooth muscle cells examination internal cervical os is dilated.
C. Embryonic rests Which would be the best treatment option:
D. Placental remnants A. D & C (dilatation & curettage) T
E. Pluripotent endometrial epithelium B. E & C (evacuation & curettage)
C. MVA
107. Polypropylene ring pessary is indicated for UV D. Hysterectomy
prolapsed if: E. D, E & C (dilatation, evacuation & curettage)
A. Patient had no bowel or bladder related
symptoms
B. Patient is suffering from viral hepatitis
C. Patient refuse for surgery
D. Prolapse is of mild degree

Page | 10
113. A 45-years old woman had abdominal D. Ergonovine maleate
hysterectomy due to menorrhagia 02 weeks ago. E. Misoprostol
She has come to gynecology clinic with
complaints of continuous leakage of urine. 118. Combined hormonal contraceptives are
Which of the following could have caused this beneficial in the reduction of the risk of which
problem: of the following:
A. Vesico-vaginal fistulas T A. Endometrial carcinoma T
B. Overflow incontinence due to postoperative B. Leomyosarcoma of uterus
pain & inflammation C. Squamous cell carcinoma of ovary
C. Detrusor instability D. Cervical adenocarcinoma
D. Urinary tract infection E. Carcinoma of breast
E. Displacement of urethra-vesical angle
119. Which one of the following uterine abnormality
114. A 28-years old woman married for five years is primarily due to failure of the fusion of only
had tubal surgery for blocked tubes 06 months upper part of mullerian ducts:
before, now presented at eight weeks of A. Septate uterus
gestational amenorrhea with lower abdominal B. Unicornuate uterus
pain and mild vaginal bleeding. On pelvic C. Arcuate uterus
examination cervical os is closed, excitation sign D. Uterus didelphys
is positive along with tenderness. Most probable E. Bicornuate uterus
diagnosis is:
A. Threatened abortion 120. A 48-years old lady para is C/O stress
B. Urinary tract infection incontinence following the birth of her first
C. Torsion of ovarian cyst child 15 years ago. She had abdominal
D. Tubal pregnancy hysterectomy due to menorrhagia 01 year back
E. Missed abortion T and since then her symptoms have become
worse. She weighs 90 kg and drinks a lot of tea
115. Luteinization occurs normally in the ovary throughout the day. Which one of the following
during each menstrual cycle. Which of the will provide her best relief:
following best describes the process: A. Reduction of weight T
A. The ovarian stroma undergoes adipose B. Reducing tea consumption
degeneration prior to ovulation T C. Burch’s colposuspention
B. The non-ovulated follicles undergo fatty D. Anterior colporaphy
degeneration E. Transvaginal tape insertion
C. The granulosa cells turn red
D. Cysts form in the theca 121. A woman who is rhesus negative undergoes
E. Mature granulosa & the theca interna cells amniocentesis at 16 weeks. To prevent iso-
become epithelioid & form a corpus luteum immunization she should take anti-D:
A. Before the procedure
116. A 33-years old patient has been diagnosed as B. After the procedure
having adenomyosis. Which of the following C. At 20 weeks
symptoms is most consistent with this D. At 34 weeks T
diagnosis: E. After the delivery of placenta
A. Infertility
B. Secondary dysmenorrhea 122. A 29-years old woman with primary fertility.
C. Mood swings Her BMI is 30. She is known to have a PCO
D. Painful defecation based anovulation. She has remained an
E. Dyspareunia T ovulatory cycle despite of increasing dose of
clomiphene citrate over 06 cycles. What is the
117. A patient complains of heavy but irregular most appropriate next step in her management
menstrual periods. An anatomic cause of the option:
magnitude of her flow has been ruled out. A. LOD (ovarian drilling) T
Which of the following has been shown to be B. ICSI
most effective in reducing rather than C. IUI
eliminating her menstrual flow: D. Metformin
A. Tranexamic acid E. IVF
B. Dilatation & curettage
C. Depot medroxyprogesterone acetate (DMPA)
Page | 11
123. During the menstrual cycle the histologic 128. A 16-years old girl brought by her mother in
appearance of the endometrium will change gynecology clinic with history of primary
significantly. During the first half of the amenorrhea. She complaining of cyclical severe
menstrual cycle the endometrium becomes lower abdominal pain for few days each month.
thicker and rebuilds largely in response to On examination her secondary sexual
which of the following: characteristics are present. The most likely
A. Follicle stimulating hormone (FSH) diagnosis is:
B. Gonadotropin releasing hormone (GnRH) A. Mullerian agenesis
C. Estrogen B. Premature ovarian failure
D. Luteinizing hormone (LH) C. Constitutional delay
E. Progesterone T D. Imperforated hymen T
E. PCO
124. A ---years old married for four years presents
with primary subfertility. Her laparoscopy 129. A 30-years old nulliparous woman conceived
reveals minimum endometriosis. The best after fertility and became amenorrhic.
treatment option will be: Ultrasound shows 05 weeks alive pregnancy on
A. Ovulation induction right adnexa. She is complaining of abdominal
B. Expectant management pain. Her BP is 110/70 mmHg and pulse rate is
C. Cauterization of endometriosis 90 beats/min. What would you do next:
D. IVF A. Admit for laparotomy
E. Danazole T B. Prepare for laparoscopy
C. Order a β-HCG report T
125. A 55-years old postmenopausal lady presents in D. Counsel for expectant management
outpatient department complaining of bloating E. Admit her for monitoring
and mild lower abdominal pain. On
examination there is a vague mass in left 130. A 15-years old girl complaining of increasing
adnexa. What should be your next advice to this hair over her face and chest. She also has
lady in evaluation of her mass: deepening of voice and clitoromegaly. Which of
A. Trans-abdominal scan the following is the best diagnostic test in her
B. CA-125 case to confirm the diagnosis:
C. Trans-vaginal scan A. Testosterone levels T
D. CT scan B. Luteinizing hormone
E. Doppler ultrasound scan C. Karyotype
D. Dexamethasone suppression test
126. Anterior vulvar cancer is most likely to spread E. 17-hydroxy progesterone levels
primarily to which of the following lymph
nodes: 131. Which is the best accepted diagnostic test for
A. Inguinal T tubal assessment in infertility:
B. Para-aortic A. Laparoscopy & dye hydrotubation
C. Obturator B. X-ray hysterosalpingography
D. Ovarian C. Hysterosalpingo contrast sonography T
E. Femoral D. Hysteroscopy
E. Laparoscopy
127. Progesterone only contraception that has been
effectively used as an alternative to 132. A 47-years old patient presented at gynecology
hysterectomy in selected cases is: clinic with severe vasomotor symptoms. Her
A. Progesterone only pill taken quarterly past history reveals breast cancer. What
B. Implanon (subdermal implant) treatment option is best for her:
C. LNG intrauterine system T A. Conjugated equine estrogen
D. Gestrenone B. SSRI (selective serotonin reuptake
E. Long acting depot progesterone inhibitor) T
C. Clonidine
D. Combined HRT
E. SNRI (serotonin & norepinephrine reuptake
inhibitor)

Page | 12
133. The best management option for blighted B. CA-125
ovum, embryonic failure and intrauterine C. Alpha fetoprotein
embryonic demise is: D. Beta HCG
A. Immediate evacuation of uterus E. TSH
B. Expectant management for 02 weeks
C. Evacuation whenever she bleed 139. A woman presents with history of overdue for
D. Left for spontaneous resolution in accordance six weeks and pain in lower abdomen you
to preference of women & consultant suspect her as having ectopic pregnancy. The
E. Medical termination of pregnancy useful test is:
A. Urine pregnancy test
134. A 38-years old African American woman B. Serum β-HCG
presents with heavy menses and an enlarged C. Serum progesterone
uterus. After an examination the clinical D. Transvaginal ultrasound T
diagnosis is leiomyoma of the uterus. Which of E. Transabdominal ultrasound
the following test describes this finding:
A. A rapidly dividing necrotic malignancy 140. What is the first line pharmacological
B. Erythematous, tender & hereditary treatment for ovulation induction in a woman
C. A soft, inter-digitating mass of the uterine with polycystic ovary syndrome (PCO):
wall T A. Anastrazole
D. A rounded, smooth, firm, well-circumscribed B. Clomiphene T
mass C. Letrazole
E. A premalignant papule of the uterine wall D. Recombinant FSH
E. Tamoxifen
135. Histologically the presence of which of the
following would determine that an ovarian 141. A 35-years old woman para 05 presents with
teratoma is malignant: history of heavy menstrual bleeding for last 06
A. Neural ectoderm months. She is anemic otherwise well.
B. Immature fetal-like cells Examination reveals normal uterus. What
C. An ovarian capsule treatment option would you offer:
D. All three germ cell line T A. Tranaxemic acid
E. Squamous cells B. Mefonamic acid
C. Combined oral contraceptive pill
136. Factors that increase the risk for carcinoma D. Progestogens
endometrium: E. LNG-IUS (levo norgestrol intrauterine
A. Multiparity system) T
B. Combined oral contraceptive pill
C. Smoking 142. A 16-years old unmarried girl presented with
D. Late menopause T irregular and heavy menstrual bleeding. She
E. Progesterone also experience severe dysmenorrhea. What is
the best medical option for her problem:
137. A 25-years old woman presents with gestational A. Combined oral contraceptive pills (COCP)
amenorrhea 08 weeks and passage of grape like B. Ethamxylate
vesicles. On examination uterus is larger than C. Goserelin
dates. The management is: D. Mefenamic acid T
A. Evacuation of uterus & follow up T E. Norethisterone
B. Counseling about follow up program
C. Registration to the anco---- unit 143. A 30-years old lady presents with secondary
D. X-ray chest infertility for 05 years. She gives history of two
E. Antibiotic episodes of PID in last 06 months. To evaluate
further she needs:
138. A 45-years old woman presents with irregular A. FSH assay
heavy vaginal bleeding. Ultrasound reveals B. FSH/LH assay
enlarged uterus with endometrial thickness of C. HSG
1.8 cm and right adnexal mass of 8x8 cm. A D. Laparoscopy & dye test T
provisional diagnosis of granulosa cell tumor is E. Hysteroscopy
made. She should have following tumor marker
to support the diagnosis:
A. Serum inhibin T

Page | 13
144. A 30-years old nulliparous woman presents in E. 47XXY
OPD with cyclical pelvic pain for last 06
months. She is on sequential OCP and NSAIDs 150. A 35-years old para 8+0 presents with post
for last 03 months. There is no improvement. coital bleeding for 06 months, clinical
What is the next most appropriate step in examination is unremarkable. What should be
management: the next diagnostic test:
A. Combined OCP for next 03 months T A. High vaginal swab
B. Consider laparoscopy for further evaluation B. Colposcopy
C. Gonadotropin releasing hormone agonist C. Colposcopic guided biopsy
D. Danazol D. Pap smear T
E. Aromatase inhibition E. Hysteroscopic guided biopsy

145. A young woman presents in outpatient clinic 151. A menopause woman presents with progressive
with hysterosalpingogram report shows dysmenorrhea and dyspareunia, she undergoes
bicornuate uterus and absent left kidney. This laparoscopy and is diagnosed as having pelvic
defect occurs due to defective development of: inflammatory disease (PID). PID is considered
A. Mesonephric ducts as a complication of:
B. Mullerian ducts T A. Gonorrhea
C. Sinovaginal bulb B. Chlamydial infection T
D. Urogenital sinus C. Mycoplasma genitalium
E. Mullerian tubercle D. Trichomoniasis
E. Bacterial vaginosis
146. Considering the risk of epithelial malignancies
in combined oral contraceptive (COC) users. 152. A 52-years old woman presents with history of
They appear to: night sweats, palpitation, anxiety and mood
A. Increase the risk of carcinoma endometrium changes. What is the likely cause:
B. Increase the risk of ovarian cancer A. Hyperthyroidism
C. Increase the risk of carcinoma of cervix T B. Hypothyroidism
D. Protect against the cancer of ovaries & C. Diabetes mellitus
endometrium D. Climacteric symptoms T
E. Protect against the cancer of cervix E. Anxiety

147. A 35-year old woman presents with abnormal 153. A woman is undergoing laparoscopy for
uterine bleeding for which hysteroscopy is chronic pelvic pain. The suitable distension
planned. She is at possible risk of: medium for diagnostic laparoscopy is:
A. Excessive hemorrhage A. Normal saline
B. Perforation of uterus T B. O2
C. Pelvic infection C. CO2 T
D. Injury to urinary bladder D. Nitrous oxide
E. Injury to gut E. A mixture of oxygen & nitrous oxide

148. Puberty is the process of reproductive and 154. A 30-years old lady presents with secondary
sexual development and maturation that amenorrhea since she had evacuation for
changes a child into an adult. First physical RPOCs 04 months ago. Her urine pregnancy
sign of puberty is: test is negative. Findings are suggestive of:
A. Pubic hair growth A. Sheehan’s syndrome
B. Axillary hair growth B. Kallman’s syndrome
C. Growth spurt C. Asherman’s syndrome T
D. Breast budding T D. Meyer Rockitnasky syndrome
E. Menstruation E. Polycystic ovarian syndrome

149. A 14-years old girl presents with primary 155. The well-established etiology for carcinoma of
amenorrhea, short stature, webbed neck. She is cervix is:
diagnosed as “Turner’s syndrome”. The A. Smoking
karyotype of this syndrome is: B. HPV infection T
A. 46XX C. Multiparity
B. 46XY D. Use of oral contraceptive pills
C. 45XO T E. Sexual intercourse
D. 47XXX
Page | 14
156. The commonest site of ectopic pregnancy is: D. Hysteroscopy
A. Ampullary region of fallopian tube T E. Hystero-salpingo-gram
B. Isthmus of fallopian tube
C. Interstitial end of fallopian tube 162. A 60-years old woman presents with large
D. Ovarian implantation pelvic-abdominal mass extending to level of
E. Cervical implantation xiphisternum, ultrasound and CT scan show
ovarian tumor. How would you manage the
157. Which form of long acting reversible patient:
contraception (LARC) is associated with delay A. Laparoscopic ovarian cystectomy
in return of fertility: B. Total abdominal hysterectomy, bilateral
A. Nexplanon salpingo-oophorectomy
B. Copper T C. Total abdominal hysterectomy, bilateral
C. Progesterone only injectable (Depot Medroxy salpingo-oophorectomy, omeutectomy &
Progesterone Acetate) T debulking of tumor deposits T
D. LNG IUS D. Chemotherapy followed by laparotomy
E. Jadelle E. Radiotherapy

158. A 60-years old lady presents with 163. A known diabetic woman presents with vaginal
postmenopausal bleeding, endometrial biopsy discharge and intense itching. She is likely to
shows carcinoma endometrium. The treatment have:
option is: A. Bacterial vaginosis
A. Chemotherapy B. Vulvovaginal candidiasis T
B. Radiotherapy C. Trichomoniasis
C. Total abdominal hysterectomy D. Gonorrhea
D. Total abdominal hysterectomy bilateral E. Chlamydial infection
salpingo-oophorectomy (BSO) T
E. Total abdominal hysterectomy with BSO, 164. A 28-years old woman para 1+4 has presented
pelvic lymphadenectomy at 16 weeks of gestation with history of 04
consecutive mid-trimester miscarriages. She
159. A 35-years old P2+0, present to you with gives history of ruptured fetal membranes prior
history of prolonged heavy periods, weight to onset of uterine contractions. The most likely
gain, fatigue and voice change. The most likely cause for her miscarriage is:
diagnosis is: A. Antiphospholipid syndrome
A. Hyperthyroidism B. Cervical incompetence
B. Hypothyroidism T C. Chromosomal abnormalities T
C. Addison’s disease D. Bacterial vaginosis
D. Adrenocortical hyperplasia E. Viral infection
E. Pituitary adenoma
165. A 25-years old woman presents with
160. A 30-years old unmarried obese woman with amenorrhea of 08 weeks and vaginal bleeding.
BMI of 30 kg/m2 presents in OPD with prolong Ultrasound shows missed miscarriage. What
menstrual cycle and excessive facial hair would you offer her:
growth. What is the first line medical A. Repeat ultrasound scan
treatment: B. Serum β-HCG
A. Combined oral contraceptive pills C. Progesterone
(COC) T D. Misoprostol T
B. Progesterone only pill (POP) E. Methotrexate
C. Cyproterone acetate
D. Metformin 166. A young 35-years old woman attend gynecology
E. GnRH analogues clinic with cervical cytology report showing
CIN-III. The next appropriate step in her
161. A 40-years old lady presents with multiple management is:
uterine fibroids, she wishes to undergo uterine A. Colposcopy
artery embolization (UAE). Which essential B. Cold coagulation
investigation needs to be carried out prior to C. Knife cone biopsy
UAE: D. Loop TZ excision T
A. Transvaginal ultrasound E. Repeat cervical smear
B. Computed tomography (CT scan)
C. Magnetic resonance imaging (MRI) T
Page | 15
167. A 56-years old woman, menopausal for two D. Pregnancy with Pyelonephritis
years had an episode of vaginal bleeding. E. Twin pregnancy
Clinical examination is unremarkable.
Transabdominal ultrasound examination 173. A 30-years old para 03 presented with 03
reveals endometrial thickness 1.2 cm. Suggest months pregnancy with mild bleeding per
the appropriate first line investigation for her: vagina since 02 days. Ultrasound report shows
A. Transvaginal sonography T dead fetus. What is the most probable
B. Endometrial sampling with pippelle diagnosis:
C. Diagnostic curettage under anesthesia A. Incomplete abortion
D. Hysteroscopic guided endometrial biopsy B. Missed abortion T
E. Pap-smear for cytology C. Inevitable abortion
D. Threatened abortion
168. Which of the following is a good indicator of E. Complete abortion
ovarian reserve:
A. Antral follicle count on transvaginal 174. A young woman has presented with secondary
ultrasound amenorrhea. On breast examination ….
B. Anti-Mullerian hormone T Present. Which one of the following hormones
C. Serum FSH may be increased on investigation:
D. Serum LH A. Estrogen
E. Serum estradiol B. Luteinizing hormone
C. Prolactin T
169. A 16-years old girl referred to gynecology clinic D. TSH (Thyroid Stimulating hormone)
by her general practitioner with primary E. Progesterone
amenorrhea. She gave history of excessive
weight gain and lethargy. Secondary sexual 175. The definition of primary amenorrhea is:
characteristics are normal on clinical A. Absence of menstruation at the age of 16
examination. Investigation reveal raised serum years in the absence of secondary sexual
thyroid stimulating hormone (TSH) and characteristics T
prolactin levels. What is the likely cause: B. Absence of menses for 06 months after
A. Pituitary prolactinoma normal cycle
B. Hypothyroidism T C. Absence of menses at the age of 14-years in
C. Hyperprolactinemia the absence of secondary sexual
D. Androgen insensitivity syndrome characteristics
E. Gonadal dysgenesis D. Absence of menses after menopause
E. Absence of menses after hysterectomy
170. With continuous long-term use of GnRH
agonists. One of the distressing side effect is: 176. Diagnosis of endometriosis is confirmed on:
A. Menopausal symptoms A. Endometrial biopsy
B. Sexual dysfunction B. Cystoscopy
C. Osteoporosis (Reduced bone mineral density C. Laproscopy T
BMD) T D. Intravenous pyelography
D. Alteration in lipid profile E. Hysteroscopy
E. Alteration in liver function test
177. Treatment of hydatidiform mole is:
171. Main support of the uterus is: A. Dilatation and evacuation
A. Broad ligament B. Hysterectomy T
B. Cardinal ligament T C. Laproscopy
C. Round ligament D. Suction, evacuation
D. Infundibulopelvic ligament E. Oxytocin infusion
E. Suspensory ligament
178. The main risk factor for causation of cervical
172. A 30-years old gravida 2nd has presented at 14 cancer is:
weeks gestation with excessive vomiting and A. Cigarette smoking
signs of preeclampsia. The uterus is larger than B. Multiparity
dates. The fetal parts are not palpable. The C. Age > 50 years
diagnosis is consistent with: D. Human papilloma virus infection T
A. Molar pregnancy T E. Herpes simplex type-II infection
B. Pregnancy with fibroid
C. Pregnancy with ovarian cyst
Page | 16
179. Endometrial carcinoma is best diagnosed by: D. Ligase chain reaction
A. Transvaginal ultrasound E. Gram staining of vaginal secretions
B. Magnetic Resonance imaging
C. CT scan 186. Common clinical presentation of patient with
D. Fractional curettage polycystic ovarian syndrome is:
E. Hysteroscopically guided endometrial A. Amenorrhea T
biopsy T B. .
C. .
180. The most common uterine malformation D. .
responsible for recurrent abortions is: E. .
A. Uterus biocornisbicollis
B. Sub-septate uterus T 187. A 47-years old Pakistani woman with BMI 35,
C. Uni-cornuate uterus BP: 140/90 mmHg was anxious about the
D. Uterus didelphys climacteric symptoms and menopausal
E. Rudimentary horn complications. What is the best advice for her
under the circumstances:
181. A 35-year old diabetic woman has presented A. Lifestyle change
with heavy menstrual bleeding for 06 B. Exercise
months…. ……..and ultrasound examination C. Calcium supplement
reveals no any abnormality; her Hb is 8.5 D. Vitamin E
gms/dl. The most appropriate treatment option E. Estrogen replacement therapy
for her is:
A. Hysterectomy 188. Oral contraceptives may increase the risk of
B. Hysterectomy & BSO following neoplasm:
C. Combined OCPs A. Breast carcinoma T
D. NSAIDs B. Ovarian carcinoma
E. LNG-IUS (Mirena) C. Endometrial carcinoma
D. Hepatic carcinoma
182. The most likely complication of cervical E. Hepatic adenoma
stenosis is:
A. Pyometra 189. An 18-years old girl presented with severe
B. Primary dysmenorrhea dysmenorrhea. The most common cause could
C. Cervical polyp be:
D. Cervical erosion A. Endometriosis T
E. Endometriosis B. Cervical stenosis
C. Partial vaginal septum
183. Which of the following germ cell tumor is D. Idiopathic
associated with ambiguous genitalia: E. Pelvic inflammatory disease (PID)
A. Choriocarcinoma
B. Gonadoblastoma T 190. The best contraceptive advice for a 38 year old
C. Dermoid cyst multiparous known hypertensive woman with
D. Carcinoid tumor BMI 32 kg/m2 is:
E. Endodermal sinus tumor A. COC pills
B. Condom
184. In the etiology of premenstrual syndrome C. Coitus interruptus
which of the following hormone has possible D. LNG-IUS T
…. E. IUCD (Copper-T)
A. Thyroxin
B. TSH 191. The modern management of menopause
C. Serotonin T comprises of:
D. Prolactin A. Low salt diet
E. FSH B. Vitamin D therapy
C. Change of lifestyle
185. A 33-years old woman is suspected to have D. Exercise
vulvovaginal candidiasis. Suggest the easily…. E. Hormonal replacement therapy T
Diagnose the problem:
A. HSV for culture & sensitivity
B. Wet mount film & microscopy
C. Candidial culture
Page | 17
192. A 30-years old sexually active nulliparous E. Simple follicular cyst
female has yellowish vaginal discharge. On
examination the cervix is erythematous and the 198. A young lady presented to you with history of
discharge reveals numerous leukocytes. Which unprotected sexual activity 08 hours ago. She
of the following is the most likely etiology: wants emergency contraception. Which is most
A. Neisseria gonorrhea suitable option for her:
B. Chlamydia trachomatis T A. COC
C. Uroplasma species B. Progesterone only pill
D. Bacterial vaginosis C. Prostinor
E. Treponema pallidum D. EC pills T
E. IUCD
193. Final physiological event of puberty in girl is:
A. Growth spurt 199. Prolapse of the anterior vaginal wall is termed
B. Breast development as:
C. Pubic hair growth A. Cystocele
D. Axillary hair growth B. Urethrocele
E. Menstruation T C. Enterocele
D. Cysto urethrocele T
194. A 46-years old P4 with complaint of severe E. Rectocele
dysmenorrhea for last 05 years and diagnosed
as a case of stage-4 endometriosis. Definitive 200. A 16-years old young girl has been admitted in
treatment option for this patient is: causality department with acute urinary
A. Bilateral salpingo oophorectomy retention. Examination reveals a cystic tender
B. Danazol mass in lower abdomen. Ultrasound showed
C. Gonadotropin releasing hormone analogues distended bladder enlarged uterus with dilated
D. Hysterectomy cavity. Vulval examination reveals blemish
E. Hysterectomy & bilateral salpingo membranes at enteroitus. The diagnosis is
oophorectomy T consistent with:
A. Imperforated hymen T
195. All are the absolute contraindications to B. Resistant ovarian syndrome
hormone replacement therapy, except: C. Testicular feminization syndrome
A. Breast cancer D. Transverse vaginal septum
B. Acute active liver disease E. Turner’s syndrome
C. Uncontrolled hypertension T
D. Migraine 201. A 38-years old multiparous woman has
E. Pregnancy presented with heavy periods. The best
management option for her is:
196. A woman of childbearing age attends gynae A. COC pills
OPD with complaint of foul smelling vaginal B. TAH
discharge around the time of menstruation. On C. Transamic acid & mefenamic acid
examination the discharge is thin, homogeneous D. Mirena T
and white in color with pH of > 4.5 microscopy E. Progesterone only pill
reveals “clue cells”. What is the most likely
diagnosis: 202. The most common organism associated with
A. Carcinoma cervix chronic PID and infertility is:
B. Candida infection A. Neisseria gonorrhea
C. Cervicitis B. Chlamydia trachomatis T
D. Foreign body C. Peptostreptococcus
E. Bacterial vaginosis T D. Treponema pallidum
E. Actinomycosis israeli
197. A 44-years old, P1+0 presents with abdominal 203. Patient with primary amenorrhea may present
mass and distension. On examination you have with:
found 4x6 form bilateral ovarian tumor. She A. Uterine didelphys
had a history of operation for gut tumor. The B. Imperforate hymen T
most likely diagnosis tumor is: C. Anorexia nervosa
A. Dermoid cyst D. Fibroids
B. Epithelial ovarian tumor E. Endometriosis
C. Germ cell ovarian tumor
D. Krukenberg tumor T
Page | 18
204. The tumor which causes primary and D. Ectopic pregnancy T
secondary amenorrhea is: E. Inevitable abortion
A. Yolk sac ovarian tumor
B. Bilateral dermoid cysts 211. Polycystic ovary syndrome is commonly
C. Pituitary adenoma T associated with:
D. Tumor of sella turcica A. Obesity T
E. Thyroid adenoma B. Frequent periods
C. Multiparity
205. While evaluating 30 years old woman with D. Galactorrhea
infertility with bicornuate uterus, additional E. Adrenal hyperplasia
testing is necessary due to increased risk of
congenital anomalies in one of the following 212. Treatment of choice for 24 years old unmarried
system: girl with BMI more than 30 and a known case
A. Skeletal of PCO is:
B. Hematopoietic A. Progesterone only pills
C. Urinary T B. COC
D. Central nervous C. Weight reduction & exercise T
E. Tracheo-esophageal D. Metformin
E. GnRH analogues
206. Which of the following finding characterizes
normal semen sample: 213. Endometriosis is a recognized cause of:
A. Agglutination A. Deep dyspareunia T
B. Sperm concentration of 35 million per ml T B. Amenorrhea
C. 05% normal sperm morphology C. Polycystic ovary syndrome
D. 10% progressive sperm motility D. Postmenopausal bleeding
E. A volume of 01 ml E. Painful laparotomy scar

207. The suitable treatment for a case of mild 214. A 42-years old para 5+0 belonging to rural area
endometriosis and infertility is: presented to you with complaint of heavy
A. Expectant menses since 02 years. Ultrasound pelvis
B. COC pills T revealed multiple fibroid on anterior and
C. Danazol posterior wall of uterus. The best treatment
D. Gestodine option for this woman would be:
E. Desogestrel A. Merina
B. Total abdominal hysterectomy T
208. A 35-years old para 0+3 first trimester C. Myomectomy
abortion. The likely cause can be: D. Uterine artery embolization
A. Uterine anomaly E. GnRH analogues
B. Cervical fibroid
C. Cervical incompetence 215. A 35-years old para 8+0 complaints of
D. Chromosomal abnormalities T dribbling of urine while coughing or laughing.
E. Anti-phospholipid syndrome She relates these symptoms since the delivery of
her last child. What is the most likely diagnosis:
209. A 24-years old para 1+0 admitted in gynae A. Detrusor instability
ward with amenorrhea 02 months and bleeding B. Overflow incontinence
per vagina. On examination uterus is 14 weeks. C. Stress incontinence T
The most probable diagnosis is: D. Vesico vaginal fistula
A. Molar pregnancy T E. Urge incontinence
B. Incomplete abortion
C. Missed abortion 216. The commonest malignant ovarian tumor in all
D. Blighted ovum ages is:
E. Complete abortion A. Sex cord stromal tumor
B. Germ cell tumor
210. A 28-years para 2+0, over due 15 days having C. Epithelial cell tumor T
severe pain in lower abdomen and fainting D. Brenner’s tumor
attacks. The suspected diagnosis is: E. Clear cell tumor
A. Missed abortion
B. Incomplete abortion
C. Hydatid mole
Page | 19
217. Operative hysteroscopy is used in all conditions, 223. Breech presentation is found in:
except: A. 3% of term gestation T
A. Adhesiolysis for Asherman’s syndrome B. 3% of preterm gestation
B. Division of uterine septum C. 5% of term gestation
C. Endometrial resection D. 5% of post-term gestation
D. Removal of theca lutein cyst T E. 1-2% of term gestation
E. Sub-mucous myomectomy
224. The confirmatory test for the diagnosis of
218. Failure rate of levonorgestrel-releasing IUCD thalassemia in a pregnant woman is:
per 100 women year is: A. Complete blood count
A. 0.5 T B. Hb electrophoresis T
B. 02 C. Absolute indices
C. 04 D. Ultrasound scan
D. 06 E. Chorion villus sampling
E. 08
225. Administration of which of the following drug
219. A 35-years old para 4+0 complaints of post to the mother will have the most significant
coital bleeding. On speculum examination beneficial effects on fetal lung maturation:
cervix is hypertrophic with ulcer. Which of the A. Dexamethasone
following investigation should be requested: B. Betamethasone T
A. Cervical biopsy C. Prolactin
B. Colposcopy D. Thyroxin
C. Endometrial biopsy E. ACTH
D. High vaginal swab
E. Pap smear T 226. The commonest symptom of ectopic pregnancy
is:
220. A 55-years old woman presents with A. Bleeding per vagina
postmenopausal bleeding, endometrium is thick B. Pain in abdomen T
on transvaginal ultrasound. Next step in her C. Dizziness
evaluation is: D. Gastrointestinal symptoms
A. MRI E. Shoulder tip pain
B. Pap smear
C. Diagnostic curettage 227. Commonly used agent for medical management
D. Pipple endometrial sampling T of ectopic pregnancy is:
E. Total abdominal hysterectomy A. KCL
B. Methotrexate T
221. A 28-years old lady presented with history of C. Mifepristone
induced abortion 02 days back followed by D. Misoprostol
severe lower abdominal pain. On ultrasound E. Prostaglandin
scan uterus was enlarged with dilated cavity
echos but uterine outline could not be 228. The tubal patency can be confirmed by all of
ascertained properly. What would be the most the following, except:
probable diagnosis: A. Ultrasound
A. Uterine perforation T B. Hysterosalpingography
B. Retained products of contraception C. Colposcopy T
C. Visceral damage D. Laparoscopy
D. Cervical trauma E. Hysteroscopy
E. Septicemia
229. Expected date of delivery is calculated by:
222. Ectopic pregnancy: A. Last menstrual period (LMP) + 12 months
A. Involves ovaries in most of the cases B. LMP + nine calendar monthly 03 months + 07
B. Involves fallopian tubes commonly T days
C. Always requires surgical treatment C. LMP + 10 calendar months
D. Should be treated medically D. LMP + 09 calendar months + 07 days T
E. Is mostly recurrent E. A & D

Page | 20
230. A young woman of 26 years G3, P2+0 comes to D. Treatment of choice is myomectomies
an obstetrician for her first antenatal visit at 36 E. All of the above
weeks gestation. She really eats fruits and fresh
vegetables. She is at increased risk of: 237. A 65-year old woman has presented with
A. Microcytic hypochromic anemia T postmenopausal bleeding for one month. She is
B. Megaloblastic anemia diagnosed as a case of carcinoma endometrium
C. Normocytic normochromic anemia stage-I. The best treatment option for her is:
D. Biphasic anemia A. Chemotherapy
E. Hereditary spherocytosis B. Radiotherapy
C. Total abdominal hysterectomy (TAH)
231. Complication of fibroid are: D. Total abdominal hysterectomy with bilateral
A. Hyaline degeneration salpingo-oophorectomy T
B. Intraperitoneal hemorrhage E. TAH plus BSO plus pelvic lymphadenectomy
C. Red generation
D. A & B 238. An 18-years old girl has presented with
E. A & C T primary amenorrhea. The clinical features are
suggestive of Turner’s syndrome. Her likely
232. A years old primigravida comes for antenatal chromosome pattern is:
checkup having 18 weeks gestation. Her CBC A. 45 XO T
report showed Hb 8gm% with decreased PCV, B. 46 XX
MCV, MCH and MCHC. What is the most C. 46 XY
probable diagnosis: D. 47 XXX
A. Iron deficiency anemia T E. 47 XXY
B. Megaloblastic anemia
C. Thalassemia 239. A young woman has presented with secondary
D. Chronic inflammatory condition amenorrhea. On breast examination
E. Sickle cell anemia galactorrhea is present. Which one of the
following hormones may be increased on
233. Mrs Laila comes for booking visit at 10 weeks investigation:
of gestation. Her last menstrual period was on A. Estrogen
7th May 2012. What is the estimated date of B. LH
delivery: C. Prolactin T
A. February 10 of the 2013 D. TSH
B. February 14 of the 2013 T E. Progesterone
C. December 10 of the 2012
D. December 14 of the 2012 240. A young sexually active woman complains of
E. December 21 of the 2012 pelvic pain not corresponding to any treatment.
In this case the most confirmatory test for
234. In complete miscarriage: establishing the diagnosis is:
A. All the products of conception present A. Hystero-salpingography
B. Os is remains always open B. Hysteroscopy
C. All the products of conception expelled C. Laparoscopy T
out T D. Trans abdominal ultrasound
D. After miscarriage severe pain persists E. Trans vaginal ultrasound
E. Always needs evaluation
241. The first sign of puberty in a girl is:
235. CTG signs suggestive of fetal compromise: A. Pubic hair growth
A. Loss of baseline variability B. Menstruation
B. Loss of acceleration C. Breast development T
C. Fetal heart rate 110-150 b/min D. Growth spurt
D. A & B T E. Axillary hair growth
E. A & C
242. The commonest cause of Asherman's syndrome
236. Regarding fibroid: is:
A. Fibroid are common being detectable A. Pelvic tuberculosis
clinically in about 20% of women over 30 B. Pelvic inflammatory disease
years of age T C. Endometriosis
B. Fibroids are independent of estrogen D. Vigorous uterine curettage T
C. Vast majority of fibroids are symptomatic E. Uterine septum
Page | 21
243. A 14-years old girl presents with heavy periods 249. An 80-years old para 8+2 came with complaint
since menarche. Ultrasound scan did not reveal of something coming out of vagina for last 05
any abnormality. The first line treatment is: years. She is a known case of diabetes and
A. Expectant uncontrolled hypertension. On pelvic
B. Mefanemic acid examination there is moderate cystocele, mild
C. Transaemic acid T rectocele and 2nd degree cervical descent.
D. Oral contraceptive pills Which of the following management option is
E. Oral hematinics suitable for her:
A. Anterior colporrhaphy
244. The gold Standard test for diagnosis of B. Lee fort operation T
endometriosis is: C. Manchester repair
A. Serum CA 125 levels D. Ring persany insertion
B. Ultrasound scan E. Vaginal hysterectomy
C. Magnetic resonance imaging
D. Laparoscopy T 250. All are the absolute contraindications to
E. CT scan hormone replacement therapy, except:
A. Suspicion of breast cancer
245. A 30-years old lady came with 03 months B. Acute active liver disease
amenorrhea and mild per vaginal bleeding with C. Uncontrolled hypertension T
lower abdominal pain for one day. Ultrasound D. Migraine
shows 12 weeks alive fetus. What will be the E. Suspected pregnancy
most probable diagnosis:
A. Complete abortion 251. Operative hysteroscopy is used in all conditions,
B. Incomplete abortion except:
C. Inevitable abortion A. Adhesiolysis for Asherman’s syndrome
D. Threatened abortion T B. Devision of uterine septum
E. Mixed abortion C. Endometrial resection
D. Removal of theca lutein cyst T
246. A para 3+0 obese lady came in OPD with E. Sub-mucous myomectomy
excessive vaginal discharge associated with
itching. Vaginal examination revealed thick 252. While evaluating a couple presenting with
curdy white discharge with red vaginal mucosa. infertility, the most important factor in
Her blood sugar is 200 mg/dl. What is the most determining the fertility is:
likely cause of discharge in this woman: A. Age of female partner T
A. Bacterial vaginosis B. Age of male partner
B. Trichomoniasis C. Duration of infertility
C. Candidiasis T D. Low number of motile healthy sperms
D. Cervicitis E. Smoking
E. Senile endometriosis
253. A woman of childbearing age attend gynae
247. Genital warts are caused by: OPD with complaint of foul smelling vaginal
A. Cytomegalovirus discharge around the time of menstruation. On
B. Herpes simplex virus examination the discharge is thin, homogeneous
C. Human immunodeficiency virus (HIV) and white color with PH of >4.5- microscopy
D. Human parvovirus B-19 reveals “clue cells”. What is the most likely
E. Human papilloma virus T diagnosis:
A. Carcinoma cervix
248. A 45-years old para 3+0 complaints of B. Candidal infection
frequency and urgency. With occasional loss of C. Cervicitis
urine. The first investigation in her case is: D. Foreign body
A. Frequency volume chart E. Bacterial vaginosis T
B. Midstream urine for culture sensitivity T
C. Random blood sugar
D. Ultrasound scan
E. Uroflowmetry

Page | 22
254. A 30-years old lady attends gynae OPD with distended bladder, enlarged uterus with dilated
complaint of amenorrhea of 03 months. cavity. Vulval examination reveals blemish
Excessive hair growth on face and markedly membranes at enteroitus. The diagnosis is
increased weight gain. She is married for past consistent with:
three years and not conceived up till now. What A. Imperforated hymen T
would be the most likely diagnosis on the basis B. Resistant ovarian syndrome
of history given by the patient: C. Testicular feminization syndrome
A. Androgen secreting tumor D. Transverse vaginal septum
B. Uterine pathology E. Turner’s syndrome
C. Polycystic ovarian syndrome T
D. Psychologically upset 260. Which of the following investigation will
E. Early pregnancy confirm the genital tract tuberculosis:
A. Histopathological examination of endometrial
255. A 44-years old, P 1+0 presents with abdominal curettage T
mass and distension. On examination you found B. Hysterosalpingography
4x6 cm fix tumor in left breast along with 14x20 C. Hysteroscopy
enlarged ovary and severe ascites. The most D. Sputum for AFB
likely tumor is: E. Ultrasonography
A. Dermoid cyst
B. Epithelial ovarian tumor 261. A 20-years old young girl has presented with
C. Germ cell ovarian tumor painful heavy periods. The best management
D. Krukenberg tumor T option for her is:
E. Simple follicular cyst A. COC pills
B. Smooth muscle relaxants
256. Mrs Ali, P 5+0 came in emergency. She has C. Ibuprofen
started OCPs sogand reception since 02 months D. Mefanemic acid T
but because of busy schedule she forgot to take E. Progesterone only pill
pill for last 02 days. Now what advice will you
give her in this situation: 262. The most common organism associated with
A. Don't worry, take your pills as usual, that's all PID and infertility is:
B. Don't worry, just take the late pill at once & A. Neisseria gonorrhea
continue as usual T B. Chlamydia trachomatis T
C. Don’t worry take most recent delayed pill, C. Poststreptococcus
discard missed pills and take extra precautions D. Treponema pallidum
for 01 week E. Actinomycosis Israeli
D. Only take extra precautions for 01 week
E. Take two pills at once & continue with 263. A 30-years old gravida 2nd has presented at 14
progesterone only pills weeks gestation with excessive vomiting and
signs of preeclampsia. The uterus is larger than
257. Mr. and Mrs. Ali came for advice on vasectomy dates. The fetal parts are not palpable. The
while discussing on risks of vasectomy. What diagnosis is consistent with:
can be the failure rate per 100 men per year: A. Molar pregnancy T
A. 0% B. Pregnancy with fibroid
B. 0.2% T C. Pregnancy with ovarian cyst
C. 0.1% D. Singleton pregnancy
D. 0.5% E. Twin pregnancy
E. 0.1%
264. A 35-years old lady has presented with
258. Prolapse of anterior vagina wall is termed as: menorrhagia. To establish the diagnosis of
A. Cystocele menorrhagia, the menstrual blood loss should
B. Urethrocele be more than:
C. Entrocele A. 80 ml T
D. Cystourethrocele T B. 40 ml
E. Rectocele C. 50 ml
D. 70 ml
259. A 16-years old young girl has been admitted in E. 60 ml
causality department with acute urinary
retention. Examination reveals a cystic tender
mass in lower abdomen. Ultrasound showed
Page | 23
265. A gravida 2nd, Para 0+1 presents with D. Pelvic inflammatory disease
gestational amenorrhea 02 months along with E. Missed abortion
mild vaginal bleeding. On examination uterine
size corresponds to dates and cervical os is 271. A young 25 years old lady attends gynae OPD
closed. Findings consistent with diagnosis of: with complaint of intense itching and soreness
A. Inevitable abortion of vulva with purulent, offensive discharge. On
B. Mixed abortion examination punctate hemorrhages seen on
C. Re-current abortion cervix the most likely diagnosis is:
D. Septic abortion A. Chlamydia trachomatis
E. Threatened abortion T B. Trichomoniasis T
C. Bacterial vaginosis
266. A 42-years old, Para 4+0 presents with D. Cervicitis
symptoms of severe menorrhagia. Ultrasound E. Physiological discharge
of pelvis reveals multiple fibroids largest 10x12
cm. Uterine cavity is almost distorted. Best 272. A 17-years old college girl presented with
treatment option for her is: severe abdominal pain for which she quit from
A. GnRH agonists routine activity during and after the
B. Hysterectomy T menstruation. The most likely diagnosis is:
C. OCC oral contraceptive pills A. PID
D. Myomectomy B. Adenomyosis
E. Uterine artery embolization C. Endometriosis T
D. Asherman’s syndrome
267. The commonest cause of anovulatory E. Endometrial polyp
subfertility in female is:
A. Hyperprolactinemia 273. Following strains of HPV causes carcinoma of
B. Hyperthyroidism cervix:
C. Hypothyroidism A. HPV 16 & 18 T
D. Polycystic ovarian syndrome T B. HPV 32 & 33
E. Premature ovarian failure C. HPV 18 & 21
D. HPV 11 & 12
268. Which of the following is an absolute E. HPV 52 & 58
contraindication of (Combined oral
contraceptive pills): 274. A 45-years old woman has presented with
A. Generalized migraine advanced ovarian cancer. The most
B. Irregular vaginal bleeding appropriate investigation to assess the extension
C. Cerebrovascular accident T of tumor to surrounding structures is:
D. Obesity A. Real time ultrasound scan
E. Diabetes B. Doppler ultrasound
C. CT scan (Computerized Tomography) T
269. A 35-years old Para 4+0, complains of post- D. MRI (Magnetic Resonance Imaging)
coital bleeding. On speculum examination there E. Laparoscopy
is cervical ectropion. Which of the following
investigation should be ordered: 275. A 25-years old nulliparous lady presents for the
A. Cervical biopsy symptoms of severe dysmenorrhea and deep
B. Colposcopy dyspareunia. On clinical examination there is
C. Endometrial biopsy nodularity of uterosacral ligaments with fixed
D. High vaginal swab retroverted uterus. Which of the following
E. Pap smear T would be the gold standard test for the definite
diagnosis:
270. A 20-years old Para 0+1 is admitted in A. MRI
emergency ward with history of overdue by 10 B. Transvaginal sonography
days and dull pain in right iliac fossa and mild C. CA-125
bleeding per vagina. Ultrasound scan reveals D. Doppler ultrasound
empty uterine cavity with right sub-ovarian E. Laparoscopy T
mass of 4x5 cm. What could be the most likely
diagnosis:
A. Unruptured tubal ectopic pregnancy T
B. Appendicitis
C. Twisted ovarian cyst
Page | 24
276. Total abdominal hysterectomy is best treatment 282. A 21-years old Para 1+0 with history of home
for: delivery presented with pelvic abscess. The
A. Young woman with heavy menstrual loss management of choice for her is:
B. Young parrous woman with pelvic A. Intravenous antibiotic T
inflammatory disease B. Laparoscopy to confirm the diagnosis
C. Woman with endometriosis C. Improvement of general health
D. Nulliparous with fibroid uterus D. Drainage of abscess
E. Woman with cervical intraepithelial lesion E. Observe for spontaneous resolution
II T
283. The gold standard for diagnosis of pelvic
277. Vigorous curettage at surgical evacuation of inflammatory disease is:
retained products of conception will result into: A. Clinical history
A. Perforation B. Clinical examination
B. Cervical trauma C. Ultrasound
C. Intrauterine adhesions T D. Culture & sensitivity test
D. Injury to large intestine E. Laparoscopy T
E. Injury to bladder
284. Regarding first trimester abortions, the
278. Commonly performed surgical procedure for commonest chromosomal abnormality found is:
urodynamic stress incontinence include: A. Triploidy raised white cell count
A. Tension free vaginal tape (TVT) T B. Monosomy raised C-reactive protein
B. Transobturator tape (TOT) C. Autosomal trisomy T
C. Burch’s colposuspension D. Tetraploidy
D. Laparoscopic colposuspension E. Balanced translocation
E. Artificial sphincter
285. The unlikely complication of chlamydia is:
279. In an old frail lady with U-V prolapse, which A. Pelvic inflammatory disease
one is most appropriate surgical choice if she B. Perihepatitis Fitz-Hugh-Curtis syndrome
cannot sustain anesthesia for long period: C. Neonatal conjunctivitis
A. Manchester’s repair D. Adult conjunctivitis
B. Lefort’s calpocleisis T E. Bartholin’s abscess T
C. Hysterosacropexy
D. Sacrocervicopexy 286. Regarding cystometry, which of the following
E. Sacrospinous ligament fixation statement best explains the test:
A. It measures intravesical pressure
280. Natural menopause is best diagnosed by: B. Measures bladder volume
A. Cessation of menses between age 40 to 45 C. Measures pressure volume relationship T
years for six months D. Measures detrusor pressure
B. Presence of hot flushes E. Cannot measure continuous bladder pressure
C. Irritability & lack of concentration
D. Cessation of menses between age 45 to 55 287. A young nulliparous woman has come with
years for 12 months T second degree uterovaginal prolapse. What is
E. Serum FSH is more than or is equal to the most suitable treatment option for her:
40iu/ml A. Kegel’s exercise
B. Manchester’s repair
281. A 52-years old lecturer postmenopausal for two C. Vaginal hysterectomy
years presented with headache and bone pains. D. Sacrohysteropexy T
She is not taking any drug. The most widely E. Sacrocervicopexy
recommended therapeutic intervention for her
is: 288. A 65-years old postmenopausal with history of
A. Calcium rich diet vaginal hysterectomy 10 years ago presents
B. Exercise with vault prolapse. The most suitable
C. Vitamin D supplements treatment option for her is:
D. Change of lifestyle A. Kegel’s exercise
E. Change of life, calcium & vitamin D B. Plastic ring pessary
supplement T C. Abdominal sacrocolpexy T
D. Sacrospinous ligament fixation
E. Colpocliasis

Page | 25
289. Combined oral contraceptive pill acts by: 295. Inhibin is a tumor marker for which of the
A. Making the endometrium atrophic following non-epithelial ovarian cancer:
B. Suppression of follicle stimulating hormone A. Endodermal sinus tumor
C. Suppression of luteinizing hormone B. Dysgerminoma
D. Changes in cervical mucus T C. Granulosa cell tumor T
E. Suppression of follicle stimulation hormone & D. Immature carcinoma
luteinizing hormone E. Mixed germ cell tumor

290. A 25-years old Para 5+0 with heavy menstrual 296. Which of the following vulval cancer has poor
bleeding came for contraceptive advice. Which prognosis:
is the most suitable option for her: A. Adenocarcinoma
A. Combined oral contraceptive pill B. Paget’s disease
B. Barrier method C. Malignant melanoma T
C. Mirena-Levonorgesterel intrauterine D. Squamous cell carcinoma
system T E. Vercuccous carcinoma
D. IUCD (intrauterine contraceptive device)
E. Progesterone pill only 297. The most appropriate way to diagnose vaginal
carcinoma is:
291. A 25-years old girl with heavy menstrual A. History of vaginal bleeding
bleeding and anemia has been diagnosed as a B. Clinical examination
case of fibroid uterus. The uterus is 16 weeks C. Examination under anesthesia
size. The most appropriate management option D. Biopsy of growth T
is: E. Histopathology of growth
A. Expectant
B. Levonorgestrel intrauterine system 298. Genetic predisposition of ovarian cancer with
C. Combined oral contraceptive pills positive Lynch-II syndrome is:
D. Gonadotropin releasing hormones analogues A. 5-10%
(GnRH) B. Less than 05%
E. Correction of anemia & myomectomy T C. Less than 01%
D. 20%
292. A 28-years Para 4+0 presents with secondary E. More than 10% T
amenorrhea of one year and hot flushes. Her
FSH level is 40 IU/ml. The symptoms are likely 299. The best medical management of 1st trimester
due to: missed abortion comprises of:
A. Asherman’s syndrome A. Syntocinon infusion
B. Polycystic ovarian disease B. Ergometrin injection
C. Premature ovarian failure T C. Misoprostol oral or vaginal
D. Thyroid abnormalities D. Prostaglandin E2 vaginal pessaries
E. Tuberculous endometritis E. Mefiprostone oral T

293. An 18-years old presents with primary 300. A Para 02 having history of 08 weeks
amenorrhea. On examination she is 05 feet 2 pregnancy has presented with heavy vaginal
inch tall with normal breasts. The pubic and bleeding and pain lower abdomen. On
axillary hairs are scanty. External genitalia examination the cervical os is open, products of
appears normal however uterus is absent. The conceptions felt in the uterine cavity, size of
most likely diagnosis in her case is: uterus is appropriate for the dates. The clinical
A. Androgen insensitivity syndrome T diagnosis is:
B. Congenital adrenal hyperplasia A. Threatened abortion
C. Mullerian abnormalities B. Inevitable abortion
D. Turner syndrome C. Incomplete abortion T
E. XY gonadal dysgenesis D. Missed abortion
E. Septic incomplete abortion
294. The first physical sign of puberty is:
A. Appearance of axillary hair 301. Development of ovarian follicle begins in
B. Appearance of pubic hair response to:
C. Breast budding T A. GnRH
D. Growth spurt B. FSH T
E. Menstruation C. LH
D. Prolactin
Page | 26
E. GH 309. A 35-years old nulliparous woman presents
with heavy menstrual bleeding. Examination
302. The incidence of recurrent abortions is: reveals a firm mass in the lower abdomen most
A. 1 % T likely diagnosis is:
B. 3 % A. Adenomyosis
C. 4 % B. Fibroid
D. 5 % C.
E. 6 % D.
E.
303. A 35-years old P4 for complaints of postcoital
bleeding. On speculum examination there is 310. The corpus luteum is essential for the survival
cervical ectropion which of the following of pregnancy during:
investigation should be requested: A. 1-4 weeks T
A. Cervical biopsy B. 1-8 weeks
B. Colposcopy C. 6-8 weeks
C. Endometrial biopsy D. 9-10 weeks
D. High vaginal swab E. 10-12 weeks
E. Pap smear T
311. Major support of the uterus is:
304. Ovarian dysgerminoma: A. Uterosacral ligaments T
A. Is a germ cell tumor T B. Broad ligament
B. Is a radioresistant tumor C. Round ligament
C. Is associated with strong family history of D. Ovarian ligament
ovarian tumor E. Mesosalpinx
D. Is a epithelial tumor
E. Is a hormone producing tumor 312. A 46-years old P4 with complaint of severe
dysmenorrhea for the last five days and
305. Contraindication of intrauterine contraceptive diagnosed as a case of stage-4 endometriosis.
device is: Definitely treatment option for this patient is:
A. History of previous ectopic pregnancy A. Bilateral salpingo-oophorectomy
B. Asthma B. Danazol
C. Diabetes mellitus C. Gonadotropin-releasing hormone analogues
D. Pregnancy T D. Hysterectomy
E. Previous cesarean section E. Hysterectomy & bilateral salpingo-
oophorectomy T
306. The hydatidiform mole is diagnosed on the
basis of: 313. In complete hydatidiform mole, the karyotype
A. Clinical presentation is:
B. Serum beta HCG level T A. 46 XY
C. Amniography B. 46 XX usually T
D. Arteriography C. Triploidy
E. X-ray D. Tetraploidy
E. Monosomy
307. Clue cell on wet mount are suggestive of:
A. Syphilis 314. Treatment of menopausal symptoms is:
B. Bacterial vaginosis T A. Hysterectomy
C. Trichomoniasis B. Radiotherapy
D. Gonorrhea C. Chemotherapy
E. Candidiasis D. Hormone replacement therapy T
E. Laparoscopy
308. The hypothalamic pituitary ovarian axis is
functionally completed at: 315. Definitive treatment of adenomyosis in
A. Early half of fetal life T perimenopausal woman is:
B. Late half of fetal life A. Surgery T
C. Immediately after birth B. Hormone therapy
D. Just before puberty C. Radiotherapy
E. At puberty D. Laparoscopy
E. Chemotherapy

Page | 27
316. The definition of abortion is: 322. An 18-years old girl is present in OPD for the
A. Expulsion of the products of conception complication of not menstruating yet. On
before 20 weeks of gestation T examination she is 04 feet 02 inches tall and
B. Expulsion of foetus with placenta upto 28 with webbed neck and wide carrying angle. The
weeks of gestation best diagnostic tool in her case is:
C. Expulsion of products of conception before 12 A. Karyotyping T
weeks of gestation B. CT scan
D. Expulsion of only foetus upto 20 weeks of C. Hormonal assays
gestation D. MRI
E. Expulsion of foetus weighing upto 1000 gm E. Ultrasound
317. A 15-years old girl presents with heavy periods 323. Syphilis is a sexually transmitted infection
since menarche. Ultrasound scan did not reveal caused by treponema pallidum which is a:
any abnormality. The first line treatment is: A. Bacteria
A. Expectant B. Fungus
B. Mefenamic acid T C. Parasite
C. Transemic acid D. Spirochete T
D. Oral contraceptive pills E. Virus
E. Oral hemanitinics
324. Increase incidence of ectopic pregnancy is
318. Single most important factor in determining found with:
fertility is: A. STD
A. Age of the female partner T B. IUCD T
B. Age of the male partner C. Increased use of OCPs
C. Age of menarche D. Assisted reproduction
D. Size of the ovary E. Tubal sterilization
E. FSH level in the mid cycle
325. A 28-years old lady came to gynaecologist
319. Select the most appropriate prognostic test in complain of oligomenorrhea, history of weight
the follow up management of epithelial ovarian gain for 06 months and U/S report shows
cancer: multiple follicles in ovaries. What will be the
A. TVS most probable diagnosis in this case:
B. CT scan A. Hypothyroidism
C. MRI B. Premature ovarian failure
D. Serum CA 125 T C. Pelvic inflammatory disease
E. Laparoscopy D. Polycystic ovarian syndrome T
E. Functional ovarian cyst
320. The main risk factor for causation of cervical
cancer is: 326. A 53-years old, multiparous woman complaints
A. Cigarette smoking of postmenopausal bleeding and offensive
B. Multiparity vaginal discharge for 03 months. Clinical
C. Age > 50 years examination revealed atrophic uterus with
D. Human papillomavirus infection T friable, ulcerated cervix, appendages are
E. Herpes simplex type II infection thickened. What would be the most likely
diagnosis:
321. The commonest presenting symptom of ectopic A. Cancer of cervix
pregnancy is: B. Cervical intraepithelial neoplasia
A. Abnormal vaginal bleeding C. Ovarian cancer
B. Pain T D. Vaginal infection
C. Syncope E. Pelvic inflammatory disease
D. Cardiovascular collapse
E. Painful defecation 327. The effects of premature menopause include all,
except:
A. Decreased cardiovascular risk T
B. Infertility
C. Vasomotor risk
D. Vaginal dryness
E. Osteoporosis

Page | 28
328. A 55-years old woman underwent a vaginal 334. The most common symptom of endometrial
hysterectomy for U-V prolapse. The most hyperplasia is:
important cause in subsequently recurrence of A. Vaginal discharge
prolapse is: B. Pelvic pain
A. Chronic cough C. Vaginal bleeding T
B. Lifting heavy weight D. Amenorrhea
C. Menopause T E. Abdominal distension
D. Poor tissue quality
E. Trauma 335. A couple presented in OPD with history of
infertility since last two years. Husband semen
329. A 30-years old lady para 03 presented with 03 analysis was advised. What is the WHO
months pregnancy with complaints of mild criterion for minimum sperm count in a normal
bleeding per vagina since 02 days. Ultrasound person:
report shows dead fetus. What is the most A. 10 million
probable diagnosis: B. 30 million
A. Incomplete abortion C. 20 million T
B. Inevitable abortion D. 40 million
C. Threatened abortion E. 70 million
D. Complete abortion
E. Missed abortion

330. A 60-years old para 8+0 known diabetic


complaints of vulval itching. On vulval
examination there is a 02 cm white lesion over
labia majora and skin looked thin and crinkled.
Your clinical diagnosis is:
A. Chancroid
B. Litchen sclerosis T
C. Paget’s disease
D. Squamous cell hyperplasia
E. Squamous VIN

331. A 16-years old girl presents with primary


amenorrhea and lack of secondary sexual
characters with anosmia. The most probable
diagnosis is:
A. Kallman’s syndrome T
B. Turner’s syndrome
C. Down's syndrome
D. Klinefelter’syndrome
E. None of above

332. Menarche usually occurs at the age of:


A. 08 & 10 years
B. 11 & 13 years T
C. 14 & 16 years
D. 17 & 18 years
E. 18 & above

333. The most common cause of abnormal uterine


bleeding in a 13 year old girl is:
A. Uterine cancer
B. Ectopic pregnancy
C. Anovulation T
D. Systemic bleeding diathesis
E. Trauma

Page | 29
1. A 25-years old primigravida came in 5. You are asked for doing active management of
emergency with decreased fetal movement. On third stage of labour. The recommended
examination she was in labour and 04 cm management include:
dilated, CTG was pathological, therefore fetal A. 10 IU oxytocin intramuscular T
blood sampling is carried out. At what PH level B. 15 IU oxytocin intramuscular
fetal compromise is confirmed: C. 05 IU oxytocin intramuscular
A. 7.25 to 7.30 D. Oxytocin infusion
B. < 7.20 T E. Ergometrine 0.5 mg intramuscular
C. < 7.30
D. < 7.25 6. Your patient's blood sugar level are very high
E. 7.20 to 7.25 and you have decided to start insulin therapy.
Which of the following you will never start in
2. Induction of labour 28 years old primigravida gestational diabetics:
with singleton pregnancy is presented to you at A. Long acting insulin T
42 weeks pregnancy. She is anxious regarding B. Regular insulin
her mode and timing of her delivery. Her C. Ultra-short acting insulin
clinical history is normal so far, her CTG is D. Short acting insulin
reactive and bishop > 5. Which of the following E. NPH
statements is appropriate for fulfilling her
concern: 7. A 22-years old G3 para 2+0 came in OPD at 12
A. Induce labour with oxytocin weeks pregnancy with a history of 02 IUDs at
B. Wait for spontaneous labour 34 weeks pregnancy. She is a known case of
C. Elective caesarean section diabetes and conscious regarding her current
D. Induction of the labour with misoprostol pregnancy. What investigation will you advise
E. Induction of labour with prostaglandin E2 T to assist her previous glycemic level:
A. HbA1c T
3. A primigravida attending the antenatal clinic B. GCT
and is concerned about brown line that C. OGTT
appeared on skin from the umbilicus to the D. FBS
symphysis pubis. She is very anxious about this, E. Sugar profile
you reassure her that this is a normal feature of
pregnancy and is known as: 8. A prenatal diagnosis without invasive testing
A. Stria gravidarum can best be made by:
B. Choloasma A. Anomaly scan
C. Stra nigricans B. CT scan
D. Linea alba C. MRI
E. Linea nigra T D. Cell free DNA T
E. Growth scan
4. A 30-years old woman presents in outpatient
department with test positive for syphilis using 9. A primigravida at 40 weeks of gestation
VDRL screening test. Which of the following is admitted for delivery. First stage of labour
definitive test for syphilis: progresses well. After 02 hours of pushing she is
A. RPR test dehydrated, uterine contractions are 02 in 10
B. Ziehl-Nelson stain minute lasting for 15 seconds. On pelvic
C. Cholesterol-lecithin-cardiolipin antigen test examination presenting part is vertex at 0
D. PCR station, sagittal suture in transverse, mild caput
E. FTA-ABS test T
Page | 30
and CTG is showing type-Il deceleration. The C. Hydrostatic method of Osulivan's T
next step of management is: D. Laparotomy
A. Caesarean section E. Manual replacement of uterus under
B. Rehydration & intravenous oxytocin T anesthesia
C. Vacuum delivery
D. Allow another hour for passive second stage 16. A woman with twin pregnancy should be
of labour referred to tertiary care:
E. Forceps delivery A. For vaginal delivery
B. For C-section T
10. Which of the following abnormality you will C. For discordant fetal growth
suspect if a pregnant woman is taking warfarin: D. For invasive testing
A. Renal agenesis E. For an early booking
B. Neural tube defect
C. Dextrocardia 17. A 24-years old primigravida came into the
D. Claw hand emergency room at 36 weeks gestation with
E. Nasal hypoplasia T complaints of headache and upper abdominal
pain. On examination her BP was 160/110
11. A baby is delivered by caesarean section at 37 mmHg with +3 urine protein. What is your
weeks of gestation. Soon after the delivery the probable diagnosis:
baby develops tachypnea. X ray chest is clear. A. Eclampsia
The condition improved after supplemental B. Chronic hypertension
oxygen for 12 hours. The likely diagnosis of this C. Hellp syndrome
case is: D. Severe preeclampsia T
A. Transient tachypnea of new born T E. Pregnancy induced hypertension
B. Birth asphyxia
C. Pneumonia 18. A 32-weeks pregnant G2 P0+1 came in
D. Surfactant deficiency antenatal clinic with 41 weeks pregnancy
E. Meconium aspiration complaining severe itching, bile acid are
elevated, she is diagnosed as a case of obstetric
12. A 28-years old multigravida presented at 30 cholestasis. The most effective medication to
weeks of gestation with CBC showing Hb% 6.0 improve her itching is:
gm/dl with no symptoms of anemia. What is the A. S-adenosyl methionine
preferable treatment option: B. Chlorphenramine
A. Parental iron + folate C. Cholestyramine
B. Parental iron D. Antenatal choral
C. Packed cells transfusion E. Ursodeoxycholic acid T
D. Oral iron T
E. Whole blood transfusion 19. While conducting an instrumental vaginal
delivery, comparing ventouse with forceps,
13. What risk factor has highest association with there is more likely chances of:
uterine rupture in woman with previous scar: A. Perineal pain at 24 hours
A. Not monitoring labour on partograph B. Delivery by cesarean section
B. Waiting till 40 weeks are completed C. Cephalohematoma T
C. Spontaneous onset of labour D. Low APGAR score at 05 minutes
D. Induction of labour with prostaglandins E2 E. Perineal & vaginal tears
E. Use of oxytocin in labour T
20. A 23-years old unbookod primigravida referred
14. A woman should be admitted to labour room from a remote area came in an antenatal clinic
when she is contracting every 05 minutes or with 34 weeks pregnancy and she is vitally
when: stable. On her examination her BMI 18, on P/A
A. Expected date has reached T her SFH 28 weeks with cephalic presentation
B. Baby is not moving FHS 130 b/min and amount of liquor is
C. She has medical issues reduced. You suspected IUGR. Now what will
D. Her water breaks be your next investigation to reach your
E. Backache increases diagnosis:
A. Umbilical artery doppler ultrasound
15. Initial management of inversion of uterus is: B. Middle cerebellar artery doppler ultrasound
A. Hysterectomy C. Biophysical profile
B. Haultans procedure D. Ultrasound for fetal wellbeing T
Page | 31
E. CTG E. Sacrum

21. A 35-years old woman presents with an 27. A multidisciplinary team should take care of
ulcerated lesion on vulva and urinary retention. the woman with previous history of:
The most likely cause is: A. Gestational diabetes
A. HSV T B. Preeclampsia
B. Candiasis C. Placenta previa
C. HPV (human papilloma virus) D. Postpartum hemorrhage T
D. Trichomonas vaginalis E. Psychiatric disorders
E. Syphilis
28. A 28-years old P2A0 just delivered 10 days
22. A woman G3 P2+0 presents to the antenatal back visited for lower abdominal and breast
clinic at 30 weeks of gestation with varicose pain. After history you did her general,
veins. What is the underlying physiological abdominal and vaginal examination. On 10 th
mechanism: postnatal day normally where should be height
A. Effect of estrogen on vascular smooth muscles of fundus:
B. Increased plasma volume A. Palpable 05 cm below the umbilicus
C. Increased cardiac output B. Palpable 07 cm below the umbilicus T
D. Decreased vascular resistance C. Palpable 03 cm below the umbilicus
E. Venous stasis caused by weight of pregnant D. Palpable at the level of umbilicus
uterus T E. No longer palpable abdominally

23. A pale looking primigravida comes to antenatal 29. A 24-years old primigravida at 39 weeks of
clinic with the symptoms of easy fatigability gestation admitted in labour room with escape
and tiredness and pedal edema, her BP is of liquor and labour pains. On examination BP
110/70 mmHg. Your best response would be: is 120/80 mmHg, temperature is 39˚C, labour is
A. Check serum albumin expedited and fetal wellbeing is monitored.
B. Refer her to consultant Which method of fetal assessment you chose for
C. Check her hemoglobin her:
D. Regular anti-natal care T A. Fetal scalp blood sampling
E. Check serum bilirubin B. Continuous internal electronic fetal
monitoring
24. A 28-years old G2 P0+1 came in antenatal clinic C. Continuous external electronic fetal
with 41 weeks pregnancy complain of reduced monitoring T
total movements, ultrasound shows AFI 10 cm, D. Inspection of amniotic fluid
her CTG although reassuring. What would you E. Intermittent auscultation
offer to her:
A. C-section 30. Before the application of forceps the cervical
B. Induction with low threshold for section T dilatation should be:
C. Membrane sweep A. Completely dilated T
D. Leave for spontaneous labour B. > 08 cm
E. Reassurance C. > 07 cm
D. > 09 cm
25. You are asked for doing active management of E. > 06 cm
third stage of labour. The recommended
management is: 31. A 30-years old Asian woman with BMI 36
A. 15 IU oxytocin intramuscular gravida 2nd P1+0, was presented at antenatal
B. Ergometrine 0.5 mg intramuscular clinic with 20 weeks gestation. Her first baby
C. 10 IU oxytocin intramuscular T was macrosomic and stillborn, her mother is
D. 05 IU oxytocin intramuscular diabetic and you are planning OGTT at 28
E. Oxytocin infusion weeks. In this scenario which of the following is
not a risk factor of GDM:
26. An important landmark of the pelvis that A. Previous macrosomic baby
determines the distance of the descent of the B. Maternal obesity
head is known as: C. Family history of diabetes
A. Ischial spines T D. Age > 30 T
B. Linea terminalis E. Asian origin
C. Sacral promontory
D. Ischial tuberosities
Page | 32
32. A 32-years old woman has undergone 38. A 20-years old primigravida attends antenatal
emergency hysterectomy due to obstetric clinic at 06 weeks gestation. She is known to
hemorrhage during her 4th pregnancy. The have sickle cell disease and her partner’s
previous pregnancies were term C-section hemoglobin is normal. What is the most
deliveries. Examination of the uterus shows the important supplement you prescribe:
placenta has invaded the myometrium. What is A. Folic acid 01 mg throughout pregnancy
the most likely diagnosis: B. Folic acid 03 mg throughout pregnancy
A. Placenta increta T C. Folic acid 04 mg throughout pregnancy
B. Placenta percreta D. Folic acid 05 mg throughout pregnancy
C. Placenta previa E. Folic acid 05 mg until 12 weeks gestation T
D. Placenta acreta
E. Placenta abruptio 39. A 28-years old known diabetic on treatment
should plan pregnancy when her HbA1c is:
33. A woman who is rhesus negative undergoes A. < 8
amniocentesis at 16 weeks. What dose of anti-D B. < 6 T
immunoglobulin should she receive C. < 11
immediately after the procedure: D. < 4
A. 500 IU E. < 10
B. 250 IU T
C. 2000 IU 40. Pregnancy is called post-term after:
D. 1000 IU A. 42 weeks T
E. 1500 IU B. 38 weeks
C. 39 weeks
34. Postpartum hemorrhage is one of the major D. 41 weeks
cause of maternal mortality. Most cost-effective E. 40 weeks
way to prevent mortality from PPH would be:
A. Using postpartum contraceptive devices 41. Vaginal breech delivery results in high
B. Avoid macrosomia by treating diabetes perinatal mortality which can be reduced by:
C. Avoid multiple caesarean deliveries A. Applying forceps to after coming head
D. Actively managing 3rd stage using misoprostol B. External cephalic version
E. Iron supplementation & use of oxytocin C. Elective caesarean delivery T
D. Continuous electronic fetal monitoring
35. A 30-years old gravida 5th para 04 delivered E. Senior obstetrician conducting delivery
vaginally, infant weight is 4.2 kg, after 02 hours
she went into PPH. Likely cause of postpartum 42. When a woman gets pregnant with fibroid
hemorrhage in her case is: uterus and presents with acute abdominal pain
A. Uterine rupture requiring hospitalization. Best considered cause
B. Cervical tears would be:
C. Vaginal tears A. Red degeneration T
D. Coagulation disorders B. Ovarian cyst accidents
E. Uterine atony T C. Urinary tract infections
D. Placental abruption
36. A 20-years old primigravida attends antenatal E. Acute appendicitis
OPD at 20 weeks pregnancy with Hb report of
07 gm/dl. What should be the next investigation 43. A primigravida underwent vaginal delivery 03
to diagnose the cause of anemia: days ago. She is complaining of fatigue, short
A. Hb electrophoresis temperedness with tearfulness. The symptoms
B. Serum iron improved after one week without any
C. MCHC + serum ferritin medication. The diagnosis is in favor of:
D. MCV + serum ferritin T A. Postpartum depression
E. Serum folate B. Postnatal pinks
C. Postpartum psychosis
37. Which of the following is not one of the D. Schizophrenia
principles of biomedical ethics: E. Postnatal blues T
A. Non-maleficence
B. Justice
C. Dignity T
D. Autonomy
E. Beneficence
Page | 33
44. A baby delivered by C-section at 32 weeks 50. Obstetric emergencies should be dealt with
because of premature rupture of membranes structured algorithm as:
and shifted to NICU and developed respiratory A. Call help  Asses airway  Check response
distress because of: B. Call for help  Check response  Turn left
A. Anesthesia drugs lateral tilt T
B. Lack of surfactant T C. Asses breathing & circulation  Immediately
C. Lack of preoperative steroids commence CPR  Call help
D. Congenital pneumonia D. Immediately commence CPR  Turn left
E. All liquor drained lateral tilt  Call help
E. Asses airway  Asses breathing &
45. A 30-years old primigravida presents in OPD circulation  Turn on left lateral tilt
for antenatal care. Her BP is 145/98 mmHg,
dipstick urinalysis shows 2+ proteinuria. Your 51. Liquor volume is decreased because of:
further management is: A. Renal artery stenosis T
A. Antihypertensive medication B. Reduced fetal blood
B. Arrange for antenatal visit after 01 week C. Fetal hypoxia
C. Admit & monitor BP every 04 hours T D. Placental infections
D. Arrange for home blood pressure E. Maternal hypertension
measurements daily
E. Admit & treat 52. Cervical cerculage to prevent preterm labour
can be applied to:
46. A well-flexed head present at outlet as: A. Pregnancy following cervical cautery
A. Occipito-posterior B. Pregnancy following cervical cone biopsy
B. Right occipito-transverse C. Over distended uterus due to multiple
C. Occipito-anterior T pregnancy
D. Left occipito-transverse D. Over distended uterus due to fibroid uterus
E. Occipito-oblique E. Painless cervical dilatation at 34 weeks
gestation T
47. Active management of 3rd stage includes:
A. Bimanual uterine massage & 10 unit oxytocin 53. Primary postpartum hemorrhage is defined as
B. Oxytocin injection at delivery of anterior blood loss more than:
shoulder A. 1000 ml within 06 hours after vaginal delivery
C. Oxytocin infusion for next 04 hours B. 500 ml within 12 hours after vaginal delivery
D. Oxytocin injection at crowning of fetal head C. 500 ml within 24 hours after vaginal
E. Uterine massage, cord traction & oxytocin delivery T
injection T D. 500 ml within 48 hours after vaginal delivery
E. 1000 ml within 24 hours after vaginal delivery
48. An elderly primigravida developed postpartum
hemorrhage due to cervical tear following 54. A 30-years old gravida 3rd para 02 admitted in
forceps delivery when she was 09 cm dilated labour room with ruptured membranes and 08
and complaining of unbearable pain. The most cm cervical dilatation. After 01 hour she is
likely reason for this complication in her case contracting 3-4 in 10 minutes, CTG is reactive
is: and on vaginal examination now she is fully
A. Not fulfilling prerequisite of forceps dilated with occipito-anterior presentation at
application T ischial spine (0 station). The most appropriate
B. First labor management is:
C. Avoiding caesarean delivery A. Caesarean section
D. Avoiding analgesia for pain B. Start active pushing
E. Old age C. Start syntocinon augmentation
D. Apply forceps for delivery
49. Initiation of breastfeeding should be carried E. Allow an hour for passive second stage T
out:
A. Within an hour of birth T 55. In a non-diabetic woman with polyhydramnios
B. When baby is hungry commonest possibility would be:
C. Within half hour of birth A. Obesity
D. Immediately after birth B. No cause
E. When milk fills breasts after 03 days C. Chorioangioma T
D. Congenitally abnormal fetus
E. Arteriovenous fistula
Page | 34
56. Some mood swings are noticed in a young 61. A pregnant lady at 08 weeks came in antenatal
primipara following birth by C-section at clinic with H/O hyperemesis gravidarum in
public hospital. You counsel the family for previous pregnancy, now complaining of
supportive care only because commonest reason nausea and vomiting twice a day, she is afraid
for these mood changes is: of her previous experience. What advice would
A. Operative delivery T you like to give her:
B. Hormonal changes A. Reassurance that severe symptoms are
C. Being Asian unlikely to recur
D. Unsupportive staff B. Treatment with corticosteroid if symptoms
E. Young age worsen
C. Treatment with prothiazine if symptoms
57. Which of the following condition is not a high worsen T
risk for preeclampsia: D. Admit in hospital
A. Diabetes mellitus E. Prescribe some IV fluid as outpatient
B. Anti-phospholipid syndrome attendance
C. Essential hypertension
D. Thyroid disease T 62. A woman at high risk of preeclampsia should:
E. Chronic kidney disease A. Stop aspirin when antihypertensives are
started
58. A 30-years old G2 P1+0 came in antenatal clinic B. Stop aspirin week before expected delivery
at 16 weeks pregnancy with H/O macrosomic C. Take 75 mg aspirin throughout pregnancy
baby in previous pregnancy, diabetes in first D. Take 75 mg aspirin from 10 to 38 weeks T
degree relative. Her BMI is 25 kg/m2. The E. Take 75 mg aspirin during 2nd trimester
appropriate investigation you would suggest for
screening of GDM is: 63. A primigravida wants elective caesarean
A. Fasting serum insulin delivery because she is intolerant to pain. The
B. Random blood sugar best analgesia for painless delivery you offer to
C. OGTT T her is:
D. HbA1c A. Water birth
E. Fasting blood sugar B. Epidural T
C. Parenteral opioids
59. According to NICE recommendations, D. Inhalational analgesia
screening for hepatitis-C should not be carried E. Transcutaneous nerve stimulation
out in:
A. HIV positive 64. A woman at 32 weeks gestation shows a
B. Intravenous drug abuser symmetrically small fetus with an estimated
C. Commercial sex worker fetal weight on 5th centile for gestation. The
D. Every pregnant women T most likely reason could be:
E. Hepatitis-B positive A. Chromosomal abnormality in fetus T
B. Maternal weight 43 kg
60. A primigravida delivered at home presents in C. Maternal drug abuse
labour room, she is vitally stable. Clinical D. Congenital syphilis in fetus
examination reveals 2nd degree perineal tear. E. Early onset preeclampsia
Which of the following best defines 2 nd degree
perineal tear: 65. A 40-years old 2nd primigravida attends
A. Injury to perineum involving muscles but not antenatal clinic at 18 weeks gestation. She is
anal sphincter T concerned about trisomy 21, because she had
B. Injury involving > 50% external anal history of Down syndrome in previous baby.
sphincter What is the best screening option for her:
C. Injury involving both external & internal A. Measurement of maternal free HCG
sphincter B. Measurement of nuchal translucency
D. Injury involving < 50% of external anal C. Triple test
sphincter D. Quadruple test T
E. Injury to skin only E. Measurement of pregnancy associated plasma
protein

Page | 35
66. A 30-years old G5 P4A0 with previous 71. A 32-years old P2+0 delivered vaginally half
uneventful deliveries has presented at 24 weeks hour back. Suddenly she went into shock with
of gestation with complain of difficulty in weak thready pulse, blood pressure 80/50
breathing and chest pain otherwise patient is mmHg. On abdominal examination fundus of
stable. Your clinical diagnosis is pulmonary uterus not palpable and on vaginal examination
embolism. First choice of investigation in this there is mass protruding from vagina. The
scenario is: cause of her collapse is:
A. Coagulation profile A. Utero vaginal prolapse
B. CT pulmonary angiography T B. Atonic uterus
C. X-ray chest C. Rupture of uterus
D. Compression duplex ultrasound D. Cervical polyp
E. V/Q scan E. Inversion of uterus T

67. A primigravida at 40 weeks of gestation 72. A 24-years old smoker in her first pregnancy
admitted for delivery. First stage of labour attend the antenatal clinic for review, at 35
progresses well. After 02 hours of pushing she is weeks of gestation with confirm date. On
dehydrated, uterine contractions are 02 in 10 abdominal examination, her SFH is 32 cm, FHS
minutes lasting for 15 seconds. On pelvic 140 b/m, you suspect the case of IUGR. Which
examination presenting part is vertex at 0 Investigation will help you in diagnosis:
station, no caput and moulding, CTG is A. Ultrasound scan for biometry
reactive. The next step of management is: B. Biophysical profile
A. Caesarean section C. X-ray chest
B. Allow another hour for passive second stage D. CTG
of labour E. Doppler ultrasound T
C. Vacuum delivery
D. Rehydration & intravenous oxytocin 73. A 35-years woman died after C-section. She
E. Forceps delivery had shortness of breath with PPH. She was in
DIC. Postmortem showed fetal hair and
68. A 30-years old gravida 3rd para 02 admitted in epithelial cell:
labour room with ruptured membranes and 08 A. Placenta abruption
cm cervical dilatation. After 01 hour she is B. DVT
contracting 3-4 in 10 minutes, CTG is reactive C. Placenta increta
and on vaginal examination now she is fully D. Amniotic fluid embolism T
dilated with occipito-anterior presentation at E. Pulmonary embolism
ischial spine (0 station). The most appropriate
management is: 74. A young primigravida recently delivered and
A. Allow an hour for passive second stage T decided to breastfeed her baby -------:
B. Apply forceps for delivery A. 03 months
C. Start syntocinon augmentation B. 04 months
D. Caesarean section C. 02 months
E. Start active pushing D. 06 months T
E. 01 month
69. Involution of uterus after delivery takes place
by a process of: 75. A 35-years old G4P3 at 18 weeks with an
A. Autolysis T unremarkable family history has a risk after
B. Degeneration sequential screening of 1.170 for Down
C. Proteolysis syndrome. You should do which of the
D. Ischemic necrosis following:
E. Dystrophy A. Advise her the risk is quite small for
aneuploidy
70. In a woman with suspected DVT during B. Offer genetic counseling
pregnancy, the best management option would C. Perform a targeted ultrasound
be: D. Offer amniocentesis
A. No treatment till diagnosis confirmed E. Offer CVS
B. Gradual compression stockings
C. Only immobilization till diagnosis is awaited
D. Low molecular weight heparin T
E. Low dose aspirin

Page | 36
76. CTG signals sever fetal compromise if trace 82. A 24-years old G2P1 woman at 39 weeks
shows: gestation presents with painful uterine
A. Tachycardia with no acceleration contractions. She also complains of dark
B. Tachycardia with early deceleration vaginal blood mixed with some mucus. Which
C. Loss of variability with late decelerations T of the following describes the most likely cause
D. Loss of variability with no acceleration of her bleeding:
E. Fetal bradycardia with 02 acceleration A. Cervical laceration
B. Placental abruption
77. Which of the following factors tends to increase C. Placenta previa
the average duration of labor: D. Bloody show
A. Decreasing size of the baby E. Vasa previa
B. Increasing parity
C. Increasing age of the mother 83. Which one is absolute contraindication of
D. Occipito-posterior (OP) position of the baby vacuum delivery:
E. Aus-------- A. Fetal scalp oedema
B. Non-vertex presentation
78. Internal inversion and extraction at term is C. Fetal prematurity < 35 weeks
indicated in which of the following: D. Fetal coagulopathies
A. Face presentation mentum posterior E. Cervix is 09 cm dilation with fetal intolerance
B. The second twin of labor
C. Transverse lie
D. Shoulder presentation in early labor 84. Fulminating preeclampsia at 35 weeks should
E. Persistent brow be treated by:
A. Diazepam
79. A 16-years old patient delivered a term infant B. Hydralazine
yesterday. She is placing the child for adoption C. MgSo4 therapy T
and is not going to breastfeed. She asks for D. Steroids for lung maturity
something to suppress location. What is the E. Delivery of fetus
simplest and safest method of lactation
suppression: 85. Umbilical cord prolapse is most likely with
A. Breast binding, icepacks & analgesics T which of the following:
B. Depo-provera A. Transverse lie
C. Bromocriptine B. Face presentation
D. Oral contraceptive pills C. Frank breach presentation
E. Delasamone D. Oblique lie
E. Complete breach presentation
80. Mrs XY is having excessive vomiting in her first
month of pregnancy. You advise her for I/V 86. A patient calls your clinic complaining of
fluids to: continued heavy vaginal bleeding. She had an
A. Give rest to her stomach uncomplicated vaginal birth 02 weeks ago of
B. Improve renal flow her second child. What is the most likely
C. Correct electrolyte imbalance. T diagnosis from the following differentials:
D. Correct hemoconcentration A. Retained placental fragments
E. Give nutrition B. Uterine atony
C. Uterine rupture
81. A primigravida has come to you for antenatal D. Coagulopathies
care. She wants to know how much vitamin D E. Vaginal lacerations
should be taken in her vitamin pill:
A. 10 mcg T 87. Double dose of folic acid should be given to
B. 06 mcg pregnant women with:
C. 04 mcg A. Thalassemia
D. 08 mcg B. Epilepsy
E. 02 mcg C. Hemolytic anemia
D. Smoking
E. Chronic renal disease

Page | 37
88. A 30-years old primigravida at 32 weeks of D. Lymphatic
pregnancy presented with pain in abdomen. E. Fomites
Her HOF of 36 weeks, multiple fetal parts are
palpable, mild contractions on a reactive CTG, 94. The perinatal death rate is defined as which of
but vaginal exam show no sign of labor. Most the following:
likely reason for her symptoms could be: A. Infant deaths younger than 01 year per 1000
A. Hydrocephalic fetus live births
B. Multiple pregnancy B. The sum of the fetal death rate & neonatal
C. Rh isoimmunization death rate per 1000 live births T
D. Hydrops fetalis C. Fetal & neonatal deaths occurring after 36
E. Polyhydramnios weeks gestation until 03 months of life
expressed per 1000 population
89. The mother is found to have sickle trait. If the D. Deaths in utero of fetuses weighing 500 gm or
father has trait, what percentage of their more per 1000 population
children will be born with sickle disease: E. Deaths in utero of fetuses weighing 1000 gm
A. 100 or more per 1000 births
B. 00
C. 25 95. A pale looking 35 years old multigravida has
D. 75 come to your antenatal clinic at 24 weeks of
E. 50 pregnancy with shortness of breath. She is
tachycardiac and with pitting edema over legs
90. Routine screening procedures at her first and feet. The commonest reason for this clinical
prenatal care visit for a 35 years old presentation is:
primigravida with an estimated gestational age A. Hypothyroidism
(EGA) of 08 weeks should include which of the B. Anemia
following: C. Covid 19 infection
A. Ultrasound D. Chest infection
B. Toxoplasma titer E. Mitral stenosis
C. Family history
D. 01 hour glucose challenge 96. A woman at 08 weeks of pregnancy presented
E. Quadruple test with excessive vomiting, loose motions,
palpitations and mild exophthalmos and slight
91. A 36-years old woman Para 4+0, BMI 35 enlargement of thyroid gland. Her HOF is ------
presents at 24 weeks of gestation with mild --- weeks uterus is felt very soft. What is the
jaundice and itching all over the body. She most likely cause:
gives history of jaundice in every pregnancy. A. Multi-fetal pregnancy
The most probable diagnosis is: B. Fibroid uterus
A. Infectious hepatitis C. Hydatidiform mole
B. Cholelithiasis D. Mistaken dates
C. Acute cholestasis of pregnancy T E. Dermoid cyst
D. Cholecystitis
E. Substance abuse 97. The risk of serious infection transmitted by
blood transfusion is greatest for which of the
92. A multigravida who is bleeding from vagina at following:
36 weeks of gestation placenta abruption can be A. Human immunodeficiency virus (HIV)
confirmed clinically by: B. Syphilis
A. CTG trace showing deceleration C. Hepatitis C
B. Tender ridge abdomen D. Hepatitis B
C. Fundal placenta on ultrasound scan E. Creutzfeidt-Jakob disease
D. Engaged presenting part
E. Retroplacental clots after delivery of placenta 98. Immediately after the completion of a normal
labor and delivery the uterus should be which
93. Puerperal infection may be spread by severe of the following:
routes. Which of the following is the most A. Firm & rounded T
common route that results in serious B. Immobile
complication of a septic thrombophlebitis: C. Boggy
A. Venous T D. Discoid
B. Arterial E. At the level of symphysis pubis
C. Direct extension
Page | 38
99. Which of the following is the most common D. Breech at +3 station, cervix completely
time period for development of postpartum dilated, membranes ruptured
mastitis: E. Vertex at +3 station, cervix +9 cm dilated,
A. 2-3 days postpartum membranes ruptured
B. 4-6 weeks postpartum
C. 3-4 weeks postpartum 105. At delivery a perineal laceration tore through
D. 3-7 weeks postpartum the skin of the fourchette, vaginal mucous
E. 1-2 weeks postpartum membrane and the fascia and perineal muscles
of the perineal body but not the anal sphincter
100. An obese woman has developed painful swollen or mucosa. This should be recorded in the
leg following delivery. She lives a sedentary life, medical record as what type of laceration:
is anemic and smokes also. This could be due A. First degree
to: B. Complete
A. Lymphedema C. Fourth degree
B. Deep vein thrombosis D. Third degree
C. Edema due to anemia E. Second degree
D. Obstetric paresis
E. Thrombophlebitis 106. The most worrisome sign or symptom of
potentially serious pathology in late pregnancy
101. A 2nd gravida has come to antenatal clinic on is which of the following:
her booking visit, she had a history of diabetes A. Nocturia
at 30 weeks of gestation in her previous B. Visual changes
pregnancy. When would you test her for C. Swollen ankles
detection of diabetes mellitus: D. Heartburn
A. 24 weeks of pregnancy E. Constipation
B. 22 weeks of pregnancy
C. 16 weeks of pregnancy 107. A 14-years old girl is seen for her first prenatal
D. 18 weeks of pregnancy visit at 34 gestation by menstrual history. On
E. 20 weeks of pregnancy examination her BP is 148/96 mmHg and her
fundus measures 33 cm. Her urine dipstick is
102. A multigravida patient has come to antenatal +1 positive for protein. Which of the following
clinic at 36 weeks of gestation with complaints is the most likely diagnosis:
of decreased fetal movements. On examination A. Mild eclampsia
her BP is 140/95 mmHg and HOF is 33 cm and B. Third trimester pregnancy
ultrasound shows AFI 14 and estimated fetal C. Chronic hypertension
weight as 02 kg. Most likely diagnosis is: D. Hypertensive disease with superimposed
A. Oligohydramnios preeclampsia
B. Premature labor E. Preeclampsia
C. IUGR
D. Preeclampsia 108. Which of the following maneuvers is useful
E. PIH when encountering shoulder dystosia:
A. Inventional fracture of fetal humerus
103. The safest, most precise and simplest method of B. Delivery of interior arm
placental localization is which of the following: C. Suprapubic pressure
A. Auscultation D. Internal podalic version
B. Soft tissue X-ray E. Fundal pressure
C. Radioisotope study
D. Ultrasonography T 109. You have just delivered an 18 years old woman
E. Abnormal palpation (gravida 1 para 0). She is preeclamptic. Her
uterus is soft with moderate to heavy bleeding.
104. Application of forceps is appropriate in which Examination reveals no laceration. You
of the following situations: diagnose uterine atony. Which of the following
A. Vertex at +1 station, cervix completely is the best management option:
dilated, membranes intact A. 10 units of oral oxytocin
B. Transverse lie, +3 station, cervix completely B. 250 ug prostaglandin F2-alpha orally
dilated, membranes ruptured C. 0.2 mg intramuscular (IM) ergonovine
C. Mentum anterior, +3 station, cervix (methergine)
completely dilated, membranes ruptured T D. 0.2 mg oral ergonovine
E. 20 units of IV oxytocin
Page | 39
110. Ultrasound examination for confirmation of (normal 5). What is the next step of
low-lying placenta is best performed at: management:
A. Dating scan A. Thyroxine therapy T
B. At 34 weeks gestation B. Repeat thyroid function test in 04 weeks
C. Anomaly scan C. Carbimazole
D. At 36 weeks gestation D. Radioactive iodine
E. At 32 weeks gestation E. Propylthiouracil

111. During late pregnancy which of the following 117. Regarding structure and functioning of
implies urinary tract disease: placenta, maternal and fetal exchange takes
A. Decreased serum creatinine place at the level of:
B. Dilatation of the ureters A. Primary & secondary stem villi
C. Decreased creatinine clearance B. Secondary & tertiary stem villi
D. Failure to excrete concentrated urine after 18 C. Tertiary villi only
hours without fluids D. Tertiary & terminal villi
E. Glucosuria E. Terminal villi only T

112. Which of the following is associated with 118. A significant factor for uterine rupture is:
preterm delivery: A. Augmentation of labour
A. Long cervical length B. Fetal version
B. Abdominal pregnancy C. Highly parity
C. Uterine fibroids D. Induction of labour
D. Prior caesarean delivery E. Uterine scar
E. Cervical dysplasa
119. A primigravida came in labour ward at 34
113. A 25-years old primigravida at 14 weeks of weeks of gestation and mild vaginal bleeding
gestation is worried of rubella infection, since for 01 hour. On examination her blood pressure
one of her neighbor had termination of was 100/60 mmHg, pulse 100 beats/min, her
pregnancy due to +ve rubella testing. At what uterus is tense and tender and FHS were
time risk to fetus is greatest with rubella irregular. What is your most probable
infection: diagnosis:
A. Pre-conceptional A. Abruptio placenta T
B. 1st trimester T B. Placenta previa
C. 2nd trimester C. Preterm labour
D. 3rd trimester D. Infection of genital tract
E. Immediately after recovery E. Vase previa

114. A pregnant woman in antenatal clinic comes 120. A 33-years old known diabetic came at 08
with a VDRL report positive for syphilis. The weeks of pregnancy with fasting blood glucose
definitive test for syphilis is: 114 mg/dl and blood pressure 130/90 mmHg.
A. Cholesterol lecithin-cardiolipin antigen test To prevent preeclampsia in her case she needs:
B. FTA abs test T A. Low dose aspirin at 08 weeks T
C. PCR B. Low dose aspirin at 12 weeks
D. RPR test C. Methyldopa
E. Ziehl-Neilson stain D. Hydralazine
E. Folic acid
115. The common pregnancy related risk factor for
developing deep vein thrombosis (DVT) is: 121. Contraindication for induction of labour is:
A. Blood group O A. Preeclampsia
B. Previous thromboembolism T B. Gestational diabetes
C. Restricted activity C. Intrauterine fetal death
D. Operative delivery D. Placenta previa T
E. Obesity E. Fetal growth restriction

116. A primigravida at 07-weeks of pregnancy with 122. When you meet a pregnant woman for history
history of partial thyroidectomy for Grave’s taking, the first thing you should do is:
disease 03 years ago comes with blood report of A. Introduce yourself & tell who you are T
thyroid function test showing TSH 8.6 IU/L B. Ask her age
C. Ask how many kids she has
Page | 40
D. Mode of delivery D. Cocadamol
E. Menstrual history E. None of the above

123. A 30-years old G3 P2+0 previous 02 caesarean 129. A female with type-I diabetes mellitus would
sections now presented at 35-weeks pregnancy like to know about a deficiency of vitamin in
with major degree of placenta previa, suddenly pregnancy that can be harmful. A deficiency of
started heavy bleeding. What will be the which vitamin can lead to teratogenic effect in
management for her: child:
A. Admit for rest & observation A. Folic acid T
B. Transfuse immediately 02 units of blood B. Vitamin B12
C. Do emergency C/S C. Thiamine
D. Maintain I/V line keep under observation D. Iron
E. Proceed for vaginal delivery E. None of the above

124. Body mass index (BMI) at booking visit is 130. A 27-years old lady has had an uncomplicated
considered as normal if it is: pregnancy so far. She came to hospital 02 hours
A. 16.5-18.9 kg/m2 ago after her water broke. The midwife is
B. 18.5-24.9 kg/m2 T looking her now. She has regular contraception.
C. 19.5-23.9 kg/m2 P.V examination revealed 02 cm dilated cervix,
D. 20-26 kg/m2 vital signs are normal. What is the stage of
E. 25-29.9 kg/m2 labour:
A. 1st stage T
125. A 30-years old primigravida attends OPD with B. 2nd stage
history of 10 weeks of gestation, severe pain in C. 3rd stage
right lumbar region and high grade fever with D. 4th stage
rigors. Ultrasound scan confirms fetal E. None of the above
wellbeing. What could be the most likely
diagnosis: 131. Which method of contraception can cause risk
A. Acute pyelonephritis T of ectopic pregnancy:
B. Cystitis A. COCP
C. Septic abortion B. IUCD
D. Incomplete abortion C. Mirena
E. Appendicitis D. POP (progesterone only pill) T
E. None of the above
126. Labour in a primigravida is said to be
prolonged when its duration is more than: 132. A pregnant woman with long term history of
A. 08 hours osteoarthritis came to antenatal clinic with
B. 10 hours complaints of restricted joint movement and
C. 12 hours T severe pain in her affected joint. What is the
D. 14 hours drug of choice:
E. 16 hours A. Paracetamol
B. Steroid
127. A complete blood count report of pregnant C. NSAID
woman shows Hb: 9 gm/dl, MCV >100/fl and D. Pethadine
peripheral smear shows right shifted E. None of the above
neutrophils. The likely diagnosis is:
A. Hemolytic anemia 133. A 24-years old lady of 18 weeks pregnant
B. Iron deficiency anemia presents with pain in her lower abdomen for
C. Megaloblastic anemia T last 24 hours. She had painless vaginal
D. Sickle cell anemia bleeding. On examination abdomen is tender,
E. Thalassemia os is closed. What is the most likely diagnosis:
A. Threatened abortion T
128. A pregnant lady came with pain in her calf B. Inevitable abortion
muscles with local rise in temperature to C. Incomplete miscarriage
antenatal clinic. What treatment should be D. Complete abortion
started: E. None of the above
A. Aspirin
B. LMWH (low molecular weight heparin) T
C. Paraceramol
Page | 41
134. A woman has numerous painful ulcers on her internal cervical os. From this U/S report what
vulva. What is the cause: type of placenta previa she has:
A. Chlamydia A. Type-I T
B. Ebstein Barr virus B. Type-II
C. Herpes simplex virus T C. Type-III
D. Trichomonas D. Type-IV
E. None of the above E. None of the above

135. A 14-years old girl presents with primary 141. Secondary arrest occurs when progress in the
amenorrhea and short stature. What is the active phase is initially good but then shows or
most likely diagnosis: stops altogether typically after cervical
A. Down syndrome dilatation in centimeters:
B. Turner syndrome T A. 04
C. Fragile X syndrome B. 05
D. Klinefelter syndrome C. 06
E. None of the above D. 07 T
E. 08
136. A 26-years old woman had amenorrhea for 10
weeks and is pregnant. She experienced 142. The incidence of hypertensive disorders
hyperemesis and now she presents with vaginal complicating pregnancies is:
bleeding. On examination uterus: 16 weeks, A. 0.5-1%
closed os. What is the most probable diagnosis: B. 2-3%
A. Thyrotoxicosis C. 5-7% T
B. Twin D. 8-10%
C. Molar pregnancy T E. More than 10%
D. Ectopic pregnancy
E. None of the above 143. Complications of induction of labour include
all, except:
137. A 34-years old woman who never had fits or A. Cord prolapse
high B.P developed fits six hours after delivery B. Hyper-stimulation of uterus
of a term healthy child. What is the most likely C. Increased risk of C-section
diagnosis: D. Less chances of PPH T
A. Eclampsia T E. Uterine rupture
B. Preeclampsia
C. Epilepsy 144. The most common etiological factor for
D. All of above placenta previa is:
E. None of the above A. Previous termination of pregnancy
B. Older primigravida
138. A 35-years old G5 is carrying twins. The best C. Previous caesarean section T
method for monitoring of fetal growth and D. Breech presentation
wellbeing is: E. Multiple gestations
A. CTG
B. Maternal reporting of fetal movements 145. An abruptio placenta is always associated with:
C. Maternal weight gain A. Apparent bleeding
D. Measurement of symphyseal fundal height B. Severe pain over the uterus T
E. Ultrasound scan T C. Faintness & collapse
D. Fetal hypoxia
139. First stage of labour describe the time: E. Intrauterine death
A. Form diagnosis of 3cm dilatation of cervix
B. From diagnosis till delivery of baby 146. The most common cause of postpartum
C. Up to 7cm till delivery of baby hemorrhage is:
D. From 3cm till delivery of baby A. Genital tract trauma
E. From diagnosis to full dilatation of cervix B. Uterine atony T
10cm T C. Labour induction
D. Injudicious use of oxytocin
140. A 30-years old P2+0 G3 at 34 weeks of gestation E. Artificial rupture of membranes
come in antenatal OPD with U/S report which
shows placenta is low lying reaching the

Page | 42
147. Postpartum hemorrhage can be prevented by: 155. First trimester fetal death is best diagnosed by:
A. Careful labour monitoring A. Negative urine pregnancy test
B. Augmentation of labour B. Decrease in serum beta HCG
C. Active management of 3rd stage of labour T C. Ultrasound scan T
D. Instrumental intervention D. Loss of fetal movement
E. Analgesia E. Loss of serum progesterone

148. Obstetric fistulae usually develops after: 156. Forceps are contraindicated in:
A. Neglected obstructed labour A. Preterm delivery
B. Caesarean section B. Brow presentation T
C. Caesarean hysterectomy C. Face presentation
D. Midline episiotomy T D. After coming head delivery in breech
E. Ventouse delivery presentation
E. Occipito-posterior position
149. A collapsed grand multiparous admitted with
obstructed labour may have: 157. Which of the following is the largest diameter
A. Fulminating sepsis of fetal head:
B. Severe preeclampsia A. Occipito-frontal
C. Rupture uterus T B. Bi-parietal
D. Antepartum hemorrhage C. Occipito-mental T
E. HELLP syndrome D. Sub-occipito-bregmatic
E. Sub-occipito-frontal
150. First trimester ultrasound is done for:
A. Diagnosing the pregnancy 158. Umbilical cord prolapse is most likely with
B. Fetal biometry which of the following:
C. Knowing fetal anomalies T A. Transverse lie T
D. Assessment of fetal wellbeing B. Face presentation
E. Uterine anomalies C. Frank breach presentation
D. Complete breach presentation
151. Second trimester ultrasound is done to assess: E. Oblique lie
A. Congenital anomalies
B. Fetal biometry 159. The fetal thyroid gland begins to produce
C. Amniotic fluid volume T thyroid hormones:
D. Biophysical profile A. At the time of conception
E. Umbilical artery Doppler B. At the end of first trimester T
C. At the start of second trimester
152. Causes of symmetrical IUGR includes: D. At the end of second trimester
A. Streptococcal infection E. At the time of delivery
B. Folic acid deficiency
C. Worm infestation 160. Oxygenated blood is returned to the fetus via
D. Chromosomal defects T the:
E. Reduced utero-placental perfusion A. Umbilical vein to the liver T
B. Umbilical arteries
153. Subjective assessment of fetal wellbeing could C. Ductus venosus
be done by: D. Ductus arteriosus
A. Cardiotocography E. Umbilical circulation
B. Ultrasound biometry T
C. Fetal kick count 161. Regarding puerperal pyrexia, the following
D. Biophysical profile statement is not true:
E. Doppler ultrasound A. Enterococci, E. coli & anaerobes are
commonly associated with puerperal
154. Causes of first trimester fetal death includes: infection T
A. Viral infection B. Is defined as temperature of 38 ͦ C or higher
B. Abruptio placentae during first 24 hours of delivery
C. Chromosomal abnormalities T C. Genital tract infection following delivery is
D. Maternal hypertension
reflection to as puerperal sepsis
E. Diabetes mellitus

Page | 43
D. Common site of infection include chest, 168. Insulin secretion in pregnancy is increased by
breast, urinary tract, caesarean section & following hormone:
perineal wound A. Progesterone
E. Chest complications are most likely to appear B. Estrogen
in first 24 hours after delivery C. Growth hormone
D. Human chorionic gonadotrophin
162. All are the features of gynaecoid pelvis, except: E. Prolactin T
A. Gynecoid pelvis is most common type of
pelvis 169. A 25-years old G2 P1 is in active labour at 42
B. Transverse diameter is larger in inlet (brim) weeks of gestation. Clinically fetus appears to
C. The transverse diameter at mid cavity is weight approximately 04 kg. After delivery of
smaller than AP diameter T the head she has shoulder dystocia. The
D. Anteroposterior diameter is larger in outlet immediate step in the management is:
E. Internal rotation of presenting part occurs at A. Delivery of posterior arm
the level of mid-cavity B. Mc Robert’s maneuver T
C. Traction on the head
163. The feature of early congenital syphilis is: D. Woods corkscrew maneuver
A. Deafness E. Zavanelli’s maneuver followed by caesarean
B. Hutchinson’s teeth section
C. Keratitis
D. Maculopapular rash T 170. The mainstay for the treatment of severe
E. Microcephaly preeclampsia is:
A. Antihypertensive drug therapy
164. The treatment of choice for syphilis is: B. Delivery T
A. Aminoglycoside C. Intensive monitoring
B. Cephalosporin D. Magnesium sulphate
C. Erythromycin E. Thrombo-prophylaxis
D. Penicillin T
E. Tetracycline 171. A 20-years old primigravida had received an
episiotomy cut during second stage of labour. It
165. Which of the following drug is teratogenic in extends through the sphincter of rectum but
humans: rectal mucosa is intact. How would you classify
A. Bromocriptine this type of tear:
B. Methyldopa A. First degree
C. Metronidazole B. Second degree
D. Nifedipine C. Third degree T
E. Diethylstilbestrol T D. Fourth degree
E. Mediolateral episiotomy
166. Regarding physiological changes in
cardiovascular system during pregnancy. The 172. The major maternal physiological change occur
cardiac output increase: in normal pregnancy is:
A. In the first trimester of pregnancy only A. Fluid retention
B. In the first trimester & continue to rise till 32 B. Increase in intracellular fluid volume
weeks of pregnancy T C. Increase in extracellular fluid volume
C. In the second trimester of pregnancy only D. Increase vascularity
D. In the third trimester of pregnancy only E. Peripheral resistance decrease
E. In labour
173. The major hemodynamic physiological change
167. Perinatal deaths include: in normal pregnancy is:
A. All still births A. Fluid retention T
B. Babies dying within 28 days of birth B. Increase in intracellular fluid volume
C. All stillbirths & babies dying in 28 days of C. Increase in extracellular fluid volume
birth D. Increase vascularity
D. All stillbirths & babies dying within 07 days E. Decrease peripheral resistance
of birth T
E. All babies dying in one year of life

Page | 44
174. Secondary arrest of labour occurs when 180. A primigravida at 29 weeks of gestation was
progress in the active phase is initially good but admitted in labour room and diagnosed as a
then shows or stops altogether typically after case of PPROM, all are the important and
cervical dilatation of: relevant investigations that should be sent,
A. 04 cm except:
B. 05 cm A. Nitrazine testing
C. 06 cm B. Pap smear T
D. 07 cm T C. Ultrasound
E. 08 cm D. Genital tract swabs
E. Urine detail report
175. Common indication for induction of labour is:
A. Twin pregnancy 181. Expectant management of placenta previa is
B. Gestational diabetes required when:
C. Hypertension during pregnancy A. Pregnancy is less than 37 weeks & bleeding
D. Postdate pregnancy T mild T
E. Maternal wish B. Off & on heavy bleeding before term
pregnancy
176. Regarding epilepsy during pregnancy, the C. Minor degree of placenta previa at 38 weeks
principle concern is: of pregnancy
A. The frequency of fits increases D. First heavy episode of bleeding at 30 weeks of
B. Need to increase the dose of antiepileptic pregnancy
drugs E. Placenta previa type-III with heavy bleeding
C. Increased risk of congenital anomaly in
baby T 182. Concealed hemorrhage with abruptio placentae
D. Increased netfall in drug concentration is characterized by:
E. Compliance of patient is poor A. Bleeding from normally situated placenta
B. Formation of large hematoma between
177. The most important parameter in detection of placenta & uterus with no obvious
intrauterine growth restriction (IUGR) is the bleeding T
accurate estimate of gestational age by: C. Abdomen tender
A. Last menstrual period D. Blood tracks between the membranes &
B. Measuring crown rump length before 12 uterine wall
weeks E. Heavy episode of bleeding with passage of
C. Measuring biparietal diameter between 12-20 clots
weeks T
D. Measuring amniotic fluid index 183. Common fetal risk with placenta previa is:
E. Measuring abdominal circumference A. Preterm delivery
B. Fetal death
178. Mainstay for the treatment of severe C. Congenital malformation
preeclampsia is: D. Abnormal presentation T
A. Antihypertensive drug therapy E. Fetal hypoxia
B. Delivery T
C. Intensive monitoring 184. While doing vaginal assessment during the
D. Prophylactic magnesium sulphate process of labour, if fetal head is felt 3cm above
E. Thromboprophylaxis the level of ischial spines that means the head is
at:
179. During obstetrical abdominal examination, the A. +1 station
area which needs to be exposed extends: B. 0 station
A. From middle of chest to pubic symphysis T C. -3 station T
B. From middle of chest to umbilicus D. -1 station
C. From costal margin to suprapubic region E. -2 station
D. From xiphisternum to umbilicus
E. From xiphisternum to mid thighs 185. Which one of the following viral infections
carries highest maternal mortality:
A. Hepatitis A virus
B. Hepatitis B virus
C. Hepatitis C virus
D. Hepatitis D virus
E. Hepatitis E virus T
Page | 45
186. Anomaly scan for the baby is best advised at D. Measurement of symphysio fundal height
what period of gestation: E. Ultrasound scan T
A. 12-13
B. 14-15 192. First stage of labour describes the time:
C. 16-17 A. From diagnosis to 3cm dilatation of cervix
D. 18-19 B. From diagnosis till delivery of baby
E. 20-22 T C. From 7cm cervical dilatation till delivery of
baby
187. The CTG finding suggestive of severe fetal D. From 3cm cervical dilatation till delivery of
hypoxia in a patient, who is in early phase of baby
labour is: E. From diagnosis to full dilatation of cervix T
A. Early decelerations
B. Fetal heart rate variability of less than 08 193. In a nulliparous woman the duration of normal
beats/min labour is:
C. Fetal tachycardia A. 06 hours T
D. Late decelerations with minimal B. 12 hours
variability T C. 08 hours
E. Variable decelerations D. 05 hours
E. 10 hours
188. A P1+0 attends OPD with history of exposure
to chickenpox at 12 weeks of gestation. She had 194. All are the possible risk factors for poor
history of vaccination at childhood. What progress in labour, except:
advice you will give to her: A. Malposition T
A. Anomaly scan at 18 to 22 weeks B. Early membrane rupture
B. Give varicella zoster IgG C. Cephalic presentation in multigravida
C. Give acyclovir D. Big baby
D. Offer chorionic villous sampling or E. Small woman
amniocentesis for fetal infection
E. Test her for varicella zoster antibody T 195. A 32-years para 2+0 both spontaneous vaginal
deliveries presented at 41 weeks of gestation in
189. A primigravida is booked at 20th weeks of outpatient department. What could be the best
gestation. Her Hb is 6gm/dl, peripheral blood management plan for her:
smear shows microcytosis, anisocytosis and A. Induction of labour T
hypochromasia. What other investigation you B. Leave for 01 week
will advise to her: C. Wait for spontaneous labour
A. Serum ferritin & TIBC T D. Take detailed history
B. Serum iron E. Urgent caesarean section
C. MCV
D. MCHC 196. A 25-years old woman has presented at
E. Stool DR antenatal clinic with history of amenorrhea 34
weeks with excessive watery vaginal discharge.
190. A 30-years old gravida 3rd, para 2+0 with 36 The abdominal examination reveals SFH of 30
weeks of gestation came in OPD with cm, cephalic presentation and liquar less than
ultrasound report showing placenta anterior normal. Speculum examination reveals pool of
but low lying covering the os. She wants to amniotic fluid in vagina. The diagnosis is
know about her mode of delivery: consistent with:
A. Spontaneous vaginal delivery A. IUGR
B. Induction of labour at 37 weeks for vaginal B. Postdate pregnancy
delivery C. Preterm (prelabour) rupture of
C. Emergency LSCS membranes T
D. Elective LSCS at 37 weeks T D. Renal agenesis
E. Elective LSCS at 39 weeks E. Bacterial vaginosis

191. A 35-years old gravida 5th is having twin


gestation. The best method for monitoring fetal
growth and wellbeing is:
A. Cardiotocography
B. Maternal reporting of fetal movements
C. Maternal weight gain
Page | 46
197. A 25-years old woman who had delivered an E. Internal rotation, descent, extension, delivery
IUD baby at home by Dai presented on 5th of body
postnatal day with high grade fever and rigors.
Her pelvic examination reveals tender boggy 203. A primigravida has presented at 26 weeks of
uterus with offensive lochia. The diagnosis is gestation with multiple painful, shallow vulval
consistent with: ulcers preceded by vesicles and dysuria. The
A. Deep vein thrombosis most likely diagnosis is:
B. Meningitis A. Chancroid
C. Pneumonia B. Herpes simplex virus T
D. Puerperal sepsis T C. HIV
E. Pyelonephritis D. Pyogenic
E. Syphilis
198. The most common inherited cause of mental
retardation is: 204. A G6 P5+0 was admitted in labour room 03
A. Down’s syndrome T hours ago. On examination she was diagnosed
B. Fragile X syndrome to have “Brow presentation”. The best
C. Klinefelter’s syndrome management plan to deliver this baby would
D. Trisomy 13 be:
E. Trisomy 18 A. C-section T
B. Wait for another 03 hours
199. All are the indications of forceps delivery, C. Instrumental delivery
except: D. Normal vaginal delivery
A. Bleeding from fetal blood sampling site E. Wait for atleast 06 hours
B. Face presentation
C. Head not fully engaged T 205. Third stage of labour is defined as:
D. After coming head of breech A. Delivery of baby but before placental
E. Delivery before 34 weeks of gestation expulsion
B. Cervix is fully dilated
200. A 35-years old primigravida is screened +ve for C. 01 hour after delivery of baby
the risk of Down’s syndrome at 11th week of D. Delivery of baby to expulsion of placenta T
gestation. Most appropriate management E. Delivery of head only
option for her is:
A. Amniocentesis 206. A 26-years old multigravida at 36 weeks of
B. Cordocentesis gestation has presented with vaginal bleeding.
C. Chorion villus sampling T On examination her uterus is tense and fetal
D. Detailed anomaly scan at 18-22 weeks heart sounds are not audible. The diagnosis is
E. Triple best at 15-22 weeks consistent with:
A. Incidental hemorrhage
201. The puerperium is period following child birth B. Placenta previa
which lasts for: C. Placental abruption T
A. 04 weeks D. Varicose vein
B. 06 weeks T E. Pregnancy with
C. 08 weeks
D. 09 weeks 207. A woman has presented at 34 weeks of
E. 12 weeks gestation with headache. Her B.P is 140/100
mmHg, urine analysis shows 2+ albumin. The
202. During process of labour, series of changes diagnosis is considered with:
occur in the position and attitude of fetus while A. Adrenal tumor
passing through the birth canal. The sequence B. Essential hypertension
with which these changes occur is: C. Preeclampsia T
A. Descent, engagement, flexion, internal D. Renal artery stenosis
rotation, delivery of body E. Renal disease
B. External rotation, engagement, internal
rotation 208. The most effective pharmacological method for
C. Engagement, restitution, delivery of fetal induction of labour in primigravida at 40 weeks
body of gestation with intact membrane is:
D. Engagement, descent, flexion, internal A. Vaginal PGE2 T
rotation, restitution, extension T B. Intracervical PGE2
C. Oxytocin infusion
Page | 47
D. Oral misoprostol B. Appearance of meconium stained liquor is a
E. Vaginal misoprostol marker of fetal compromise T
C. Auscultation of fetal heart sound by Pinard’s
209. A multiparous hypertensive woman in her 6th fetoscope is convenient
pregnancy at 34 weeks presents with vaginal D. Maternal perception of fetal movements is
bleeding accompanied with abdominal pain. reliable
The most likely diagnosis is: E. Biophysical profile scoring is always required
A. Placenta previa II
B. Placenta previa IV 215. Secondary arrest is defined as:
C. Vasa previa A. Cessation of cervical dilatation following
D. Placental abruption T normal period of active phase dilatation T
E. Marginal APH B. Delay in dilatation of cervix for 03 hours
C. Delay in dilatation of cervix for 06 hours
210. A para 1+0 presented in labour room in active D. Poor progress in active phase of labour
phase of labour. Key management protocol for E. Poor progress in latent phase of labour
first stage monitoring includes:
A. Continuous emotional support to mother 216. The most suitable position of fetus for vaginal
B. Maternal monitoring by taking vitals breech delivery is:
C. Fetal monitoring by auscultation A. Extended breech with hip flexion & knee
D. Adequate pain relief extension T
E. Maternal & fetal monitoring by plotting B. Flexed breech with flexion at hip & knees
partograph T C. Footling breech
D. Knee presentation
211. Regarding female pelvis and fetal skull, one of E. One leg flexed & other leg is extended
the following statement is more appropriate:
A. The pelvic inlet is wider in the transverse 217. During labour when submento-bregmatic
plane diameter presents. The presentation is said to
B. The pelvic outlet is wider be:
C. The pelvic outlet is wider in the A. Face T
anteroposterior than in the transverse diameter B. Chin
D. Pelvic measurements change during labour C. Cephalic
E. The degree of internal rotation of fetal skull D. Breech
determine the diameter of the fetal skull E. Normal
presenting pelvis
218. Regarding shoulder presentation in labour:
212. The most common cardiotocograph finding A. The incidence of shoulder presentation in
seen in patients with obstructed labour is: labour is 1 in 25
A. Early deceleration & rising baseline rate B. Labour should be carefully monitored with the
B. Late deceleration & decreasing baseline onset of regular contraction as transverse lie
rate T can be corrected to a longitudinal lie
C. Late deceleration & rising baseline rate C. External cephalic version should be done to
D. Variable deceleration & rising baseline rate correct the lie of fetus
E. Variable deceleration & decreasing baseline D. Caesarean section should be performed to
rate avoid injury to the fetus or uterus T
E. While performing caesarean section lower
213. A gravida third para 02 with previous vaginal segment should always be opened by midline
deliveries has shown delayed progress in active vertical incision
1st stage of labour with poor uterine
contractions. The management of choice will 219. Which is the common clinical diagnosis for
be: placenta previa:
A. Augmentation with oxytocin infusion A. Th--- miscarriage in early pregnancy p----- the
B. Amniotomy with oxytocin infusion T bleeding due to placenta previa
C. Amniotomy B. Initial bleeding occurs prior to the 36 weeks
D. Wait & watch policy gestation
E. Caesarean section C. Abdominal pain occurs sometimes after 36
weeks gestation
214. Regarding intrapartum fetal monitoring: D. Fetal malpresentation is significant
A. CTG is the reliable test any other than vertex differentiating feature T

Page | 48
E. Vaginal examination in theatre with or C. Cocaine T
without anesthesia can be useful D. Cannabis
E. Heroin
220. Placenta previa can lead to high level maternal
risk due to: 226. A 36-years old woman Para 4+0, BMI 35
A. Postpartum hemorrhage T presents at 24 weeks of gestation with mild
B. Placenta accreta jaundice and itching all over the body. On
C. Air embolism examination murphy sign is positive. What
D. Postpartum sepsis could be the probable diagnosis:
E. Increased recurrence rate A. Infectious hepatitis
B. Cholelithiasis
221. A multigravida woman comes in emergency C. Acute cholestasis of pregnancy T
with history of 34 weeks gestational D. Cholecystitis
amenorrhea, bleeding per vagina, pain in lower E. Substance abuse
abdomen. On examination her blood pressure
was 140/100 mmHg, she is looking anemic. 227. Dose of folic acid in pregnant lady should be:
Ultrasound examination report shows mild A. 0.4 mg T
abruption. Which is the best management B. 0.5 mg
option: C. 0.6 mg
A. Immediate delivery D. 0.7 mg
B. Induction of labour at 37-38 weeks of E. 0.8 mg
gestation
C. Hospitalization, assessment of the maternal & 228. A young primigravida at 20 weeks of gestation
fetal condition comes to the antenatal clinic with a report of
D. Close monitoring of fetal & maternal serological test negative for immunoglobulins
condition & prolongation of gestation till IgG and IgM against rubella. You will advise
delivery her rubella vaccination:
E. Steroid administration, improving general A. At this visit
condition control of hypertension & B. At 28 weeks of this pregnancy
delivery T C. In last trimester of pregnancy
D. After pregnancy T
222. Medical conditions that unlikely increase the E. In subsequent pregnancy
risk of preeclampsia include all, except:
A. Preexisting hypertension 229. Primigravida at 20 weeks gestation comes to
B. Preexisting renal disease you with diagnosed case of toxoplasmosis. The
C. Preexisting diabetes best treatment to reduce the incidence of
D. Antiphospholipid antibodies transplacental infection is:
E. Chronic anemia T A. Spiramycin for 03 weeks T
B. Spiramycin for 05 days
223. Regarding multiple gestation, the commonest C. Macrolids for 03 weeks
type of twining is: D. Macrolids for 05 days
A. Monoamniotic & monochromic E. Spiramycin for 01 week
B. Diamniotic & monochorionic
C. Diamniotic & dichorionic T 230. Gravida 02 Para 1+0 comes to you with
D. Conjoined twins ultrasound report showing 30 weeks
E. Acardiac twins microcephaly. The most common organism is:
A. Cytomegalovirus T
224. The main etiological factor for causing B. Toxoplasmosis
preeclampsia is: C. Herpes simplex virus
A. Damage to vascular endothelial cells T D. Syphilis
B. Factor X overproduction E. Hepatitis B
C. High serum concentration of free radicles
D. Altered catecholamine homeostasis 231. A 37-years old multiparous woman diagnosed
E. A low dietary calcium as a case of hyperthyroidism at 28 weeks of
pregnancy. ----- for advice. The treatment of
225. Risk of placenta abruption is more in following choice is to use:
substance abuser: A. Radioactive iodine
A. Alcohol B. Carbamazole
B. Opiates C. Propylthiouracil T
Page | 49
D. Betamethasone B. Pulmonary edema
E. Propranolol C. Respiratory distress syndrome
D. Respiratory tract infection
232. Regarding epilepsy in pregnancy, the frequency E. Congestive cardiac failure T
of seizures increase in:
A. All pregnant women 239. Hyperemesis gravidarum predisposes women to
B. 1/3rd of pregnant women T develop Wernicke’s encephalopathy secondary
C. 50% of pregnant women to deficiency of:
D. 20% of pregnant women A. Pyridoxine (Vitamin B6)
E. 10% of pregnant women B. Thiamine (Vitamin B1) T
C. Folic acid
233. A 28-years old gravida 03 para 02 at 36 weeks D. Fluid & electrolytes
of pregnancy presented with autoimmune E. Vitamin A
thrombocytopenic purpura with platelet count
<50x109/IL. 1st line treatment is: 240. The main clinical approach for detecting small
A. Platelet transfusion for gestational age (SGA) babies in the low risk
B. Splenectomy population is by:
C. Corticosteroid therapy A. Uterine artery Doppler
D. Intravenous immunoglobulin T B. Ultrasound biometry T
E. All of the above C. Biochemical measurement
D. Measurement of symphyseal fundal height
234. The confirmatory test for the diagnosis of E. Cardiotocography
thalassemia in a pregnant woman is:
A. Complete blood count 241. A healthy primigravida presented in labour
B. Hb electrophoresis T ward at 40+ weeks of gestation. The most
C. Absolute red cell indices favorite mechanical method for intracervical
D. Amniocentesis induction of labour is:
E. Chorion villous sampling A. Intracervical Foley’s catheter
B. Laminara tent
235. Uterus comes to lie out of the pelvis at C. Membrane sweeping T
gestational age of: D. Hygroscopic dilators
A. 10 weeks E. Acupuncture
B. 12 weeks T
C. 14 weeks 242. Successful induction of labour depends on:
D. 16 weeks A. Parity of woman
E. 17 weeks B. Gestational age
C. Method of induction
236. Postpartum hemorrhage is defined as blood loss D. Bishop score T
in excess of: E. Baseline uterine activity
A. 500 ml T
B. 600 ml 243. A primigravida woman suffered from
C. 800 ml intrauterine fetal death following intrauterine
D. 1000 ml growth restriction. Past medical history is
E. 1200 ml insignificant except the history of fever, sore
throat and rashes in first trimester. What
237. The most common cause of postpartum would be the most likely diagnosis:
hemorrhage is: A. Rubella infection T
A. Retained placenta B. Parvovirus B19 infection
B. Uterine atony T C. Syphilis infection
C. Uterine rupture D. Group B streptococcus infection
D. Uterine inversion E. Haemophilus influenza infection
E. Cervical tears
244. A known diabetic patient at 35th week of
238. The use of β2 sympathomimetic drugs along gestation came for antenatal care. She is on
with steroids and fluid overload in woman with insulin for last 02 years. The best method to
multiple pregnancy for the postponement of evaluate her glycemic state is:
preterm labour can lead to life threatening A. Blood glucose profile
complications such as: B. Fasting blood glucose level
A. Pulmonary embolism C. HbA1C estimation T
Page | 50
D. Postprandial blood glucose level examination mother is hemodynamically stable
E. Preprandial blood glucose level and FHR is regular. What is your most likely
diagnosis:
245. The cardiac problem associated with very high A. Abruptio placenta
perinatal mortality is: B. Fundal placenta
A. Atrial septal defect C. Heavy show
B. Eisenmenger’s syndrome T D. Placenta previa T
C. Mitral regurgitation E. Scar dehiscence
D. Mitral valve prolapse
E. Ventricular septal defect 251. While being posted in obstetrics ward, you have
learnt to calculate the expected date of delivery.
246. Oligohydramnios is diagnosed on ultrasound Expected date of delivery calculated from LMP
when the maximum vertical pool of amniotic in days by Naegele’s is:
fluid is: A. 240
A. < 02 cms T B. 250
B. < 03 cms C. 260
C. < 04 cms D. 270
D. < 05 cms E. 280 T
E. <06 cms
252. The placenta is a feto-maternal organ.
247. The most commonly used preventive therapy in Maternal and fetal exchange takes place at:
patients with high risk of preeclampsia is: A. Cotyledons
A. Fish liner oil B. Primary villus stem
B. Low dose aspirin T C. Secondary stem villi
C. Methyldopa D. Tertiary stem villi
D. Nifedipine E. Terminal villi T
E. Strict monitoring of blood pressure
253. Secondary arrest occurs when progress in the
248. A 28-years old lady para 1+1 presented in OPD active phase is initially good but then slows or
with 05 months amenorrhea and mass in stops altogether typically after cervical
abdomen for 05 months. Mass has increased in dilatation in centimeters:
size. She is also complaining of pain in A. 04
abdomen. On abdominal examination there was B. 05
a cystic mass filling the upper abdomen, C. 06
extending from right hypochondrium to left D. 07 T
iliac fossa. Height of uterine fundus was 20 E. 08
weeks. Her ultrasound reports show cystic mass
with double membrane and septa. What is the 254. Complications of induction of labour include
diagnosis: all, except:
A. Hydatid cyst of liver A. Cord prolapse
B. Ovarian cyst B. Hyper stimulation of uterus
C. Pseudocyst of pancreas C. Increased risk of C-section
D. Dilated bowel loops D. Less chances of PPH T
E. Mesenteric cyst T E. Uterine rupture

249. All are the possible indications for classical 255. A 28-weeks pregnant, P 5+0 admitted in
caesarean section, except: maternity ward with Hb% 5g/dl, her blood film
A. Postmortem caesarean section shows microcytic hypochromic anemia. Hb%
B. Large cervical fibroid T electrophoresis test is normal. The most
C. Transverse lie confirmative test for iron deficiency anemia is:
D. Fetal distress / Prolapsed cord A. History
E. Placenta previa with large vessel on lower B. Megaloblasts on blood film
segment C. Hb% electrophoresis
D. Sickledex test
250. A 30-years old gravida 3rd, P 1+1 with last baby E. Iron studies T
born by C/S due to fetal distress, came in
emergency at 32 week of pregnancy with
history of painless vaginal bleeding with
passage of clots, which is now ceased. On
Page | 51
256. The most dynamic test for monitoring the ago. She complains of having headache, fever
growth restricted fetus is: and rigors, abdominal discomfort with diarrhea
A. Symphysio-feral height measurement and vomiting. She also complains of having
B. Fetal biometry very offensive vaginal discharge also. The most
C. Doppler scan T likely diagnosis is:
D. Fetal cardiotocography A. Postpartum eclampsia
E. Biophysical profile B. Puerperal pelvic infection T
C. Gastroenteritis
257. A 36-years old multiparous woman was D. Malaria
admitted in emergency with history of home E. Puerperal psychosis
delivery 11 days back, fever and fits. She was
agitated, restless and disoriented to 263. Use of first trimester ultrasound is especially
surroundings. She was normotensive with fever recommended to:
100 ͦF and neck rigidity was positive. Uterus A. Diagnose missed miscarriage
was involuted with normal looking lochia. B. Diagnose early pregnancy disorders
What would be the most likely diagnosis: C. See the fetal development
A. Meningitis T D. Date the pregnancy T
B. Puerperal psychosis E. Diagnose ectopic pregnancy
C. Eclampsia
D. Septicemia 264. A 30-years old lady para 1+3 came at antenatal
E. Puerperal pyrexia clinic with history of 20 weeks gestational
amenorrhea, excessive vaginal discharge and
258. The incidence of venous thromboembolic mild pain in lower abdomen. Discharge is not
complications during pregnancy is: associated with itching. She also gave history of
A. Five times greater T 2nd trimester abortion. On examination
B. Ten times greater discharge is thin white with offensive smell.
C. Twelve times greater What is the likely diagnosis:
D. Fifteen times greater A. Candidiasis
E. Twenty times greater B. Trichomoniasis
C. Bacterial vaginosis T
259. A primigravida presents at 6th week of gestation D. Cervicitis
with painless genital ulcer. On investigation she E. Atrophic vaginitis
was diagnosed to have primary syphilis. The
drug of choice for her is: 265. A 25-years lady presented with history of
A. Erythromycin induced abortion two days back, followed by
B. Penicillin T severe lower abdominal pain. On ultrasound
C. Quinolones scan uterus was enlarged and cavity was
D. Sulphadiaxine dilated, however uterine outline could not be
E. Tetracycline ascertained properly. What would be the most
probable diagnosis:
260. Basic needs of a baby following delivery are: A. Uterine perforation
A. Protection, breathing & weighing B. Retained products of conception
B. Protection & keeping warm C. Visceral damage
C. Protection, breathing, keeping warm & D. Cervical trauma
feeding T E. Septicemia
D. Protection, breathing & keeping warm
E. Cord care, eye care & weighing 266. A 16-years young girl has been admitted in
emergency department with acute urinary
261. A 35-years old primigravida has presented at retention. Examination reveals a cystic tender
36 weeks with vaginal bleeding. The initial step mass in lower abdomen. Ultrasound showed
in managing her is: distended bladder enlarged uterus with cavity
A. Administer oxygen by mask vulval examination reveals blemish membrane
B. Clear airway at entroitus. The diagnosis is consistent with:
C. Insert catheter to check urinary output A. Imperforated hymen T
D. Secure I.V line T B. Resistant ovarian syndrome
E. Turn her on side C. Testicular feminization syndrome
D. Transverse vaginal septum
262. A 30-years old lady came to OPD with history E. Turners syndrome
of normal vaginal delivery at home two days
Page | 52
267. A male presents with the infertility clinic with malignant ovarian cells. What is the stage of
the following report of semen analysis. Report this tumor:
shows volume 05, sperm count 20 million/ml, A. 1b
morphology 14% of normal forms, motility B. 1c T
56%. The features are consistent with the C. 11a
diagnosis of: D. 11b
A. Astheno-spermia E. 11c
B. Normozoo-spermia T
C. Oligozoo-spermia 273. A 30-years old para 3+0 all babies delivered by
D. Oligo-astheno-teratozoo-spermia caesarean section due to breech presentation.
E. Teratozoo-spermia Which one of the following congenital genital
tract abnormalities is a most likely cause for
268. Failure rate of levanorgestret releasing IUD per recurrent breech presentation:
100 women years is: A. Arcuate uterus
A. 0.5 T B. Bicornuate uterus T
B. 2 C. Longitudinal vaginal septum
C. 4 D. Transverse vaginal septum
D. 6 E. Uterus didylphis
E. 8
274. Secondary arrest of labour occurs when
269. A 13-years old girl is brought by her mother in progress in the active phase is initially good but
emergency with severe lower abdominal pain. then slows or stops altogether typically after
On examination she is hemodynamically stable cervical dilatation of:
but there is a solid and cystic mass in lower A. 04 cm
abdomen. Emergency U/S shows an ovarian B. 05 cm
tumor with solid and cystic parts along with C. 06 cm
some teeth and bony elements in it. The most D. 07 cm T
likely diagnosis is: E. 08 cm
A. Choriocarcinoma
B. Endodermal sinus tumor 275. The major hemodynamic physiological change
C. Embryonal cell tumor in normal pregnancy is:
D. Dysgerminoma A. Fluid retention T
E. Teratoma (dermoid cyst) T B. Increase in intracellular fluid volume
C. Increase in extracellular fluid volume
270. A young woman has presented with secondary D. Increased vascularity
amenorrhea. On breast examination E. Decrease peripheral resistance
galactorrhea is present. Which one of the
following hormone may be increased on 276. The incidence of hypertensive disorders
investigation: complicating pregnancies in United Kingdom
A. Estrogen is:
B. LH A. 0.5-1%
C. Prolactin T B. 2-3%
D. TSH C. 5-7% T
E. None of the above D. 8-10%
E. More than 10%
271. Operative hysteroscopy is used in all conditions,
except: 277. Complications of induction of labour include
A. Adhesiolysis for Asherman’s syndrome all, except:
B. Division of uterine septum A. Cord prolapse
C. Endodermal resection B. Hyper-stimulation of uterus
D. Removal of theca lutein cyst T C. Increased risk of caesarean section
E. Sub-mucous myomectomy D. Less chance of PPH T
E. Uterine rupture
272. A 40-years old woman was operated for pelvic
mass. Her findings on laparotomy were 278. The principle concern related to epilepsy in
suggestive of malignant ovarian tumor pregnancy is:
involving both ovaries with ruptured capsule A. Deficiency of vitamin K dependent factors
and ascitic fluid which was positive for B. Difficult monitoring of drug levels
C. Increased risk of congenital anomalies T
Page | 53
D. Net fall in drug concentration D. By rotating & pulling cord downward
E. Poor compliance of patient E. By asking the patient to cough

279. A 25-years old woman has presented on 5th 285. Mainstay for the treatment of severe
postnatal day with malaise, headache and fever preeclampsia is:
with rigors. Her pelvic examination reveals A. Antihypertensive drug therapy
tender boggy uterus with offensive lochia. The B. Delivery T
diagnosis is consistent with: C. Intensive monitoring
A. Deep vein thrombosis D. Prophylactic magnesium sulphate
B. Meningitis E. Thromboprophylaxis
C. Pneumonia
D. Puerperal sepsis T 286. The puerperium is period following child birth
E. Pyelonephritis in which systems of the body returns to its pre-
pregnant state and its duration is:
280. The most important parameter in detection of A. 04 weeks
intrauterine growth restriction (IUGR) is the B. 06 weeks T
accurate estimate of gestational age by: C. 08 weeks
A. Last menstrual period D. 09 weeks
B. Measuring crown rump length before 12 E. 12 weeks
weeks
C. Measuring biparietal diameter between 12-20 287. During obstetrical abdominal examination, the
weeks T area which needs to be exposed extends:
D. Measuring amniotic fluid index A. From middle of chest to pubic symphysis T
E. Measuring abdominal circumference B. From middle of chest to umbilicus
C. From umbilicus to pubic symphysis
281. The most common inherited cause of mental D. From xipihisternum to umbilicus
retardation is: E. From xiphisternum to mid thighs
A. Down’s syndrome T
B. Fragile X syndrome 288. A primigravida at 29 weeks of gestation was
C. Klinefelter’s syndrome admitted in labour room and diagnosed as a
D. Trisomy 13 case of PPROM, all are the important and
E. Trisomy 18 relevant investigations that should be sent,
except:
282. A 30-years old primigravida attends OPD with A. Nitrazine testing
history of 10 weeks of gestation, severe pain in B. Pap smear T
right lumbar region and high grade fever with C. Ultrasound
rigors. Ultrasound scan confirms fetal D. Genital tract swabs
wellbeing. What could be the most likely E. Check vital (Pulse, B.P, Temperature)
diagnosis:
A. Acute pyelonephritis T 289. A para 2+2, BMI 15, smoker has attended the
B. Cystitis OPD in late 2nd trimester. What problem she is
C. Septic abortion prone to develop in this pregnancy:
D. Incomplete abortion A. Urinary tract infection
E. Appendicitis B. Diabetes
C. Preterm labour T
283. While doing vaginal assessment during the D. IUGR
process of labour, if fetal head is felt at the level E. Preeclampsia
of ischial spines, that means the head is at:
A. +1 station 290. A primigravida after her pelvic assessment is
B. 0 station T diagnosed to have android pelvic. What is the
C. At intoitus most likely problem that she will encounter
D. -1 station during labour:
E. -2 station A. Occipito-posterior position
B. Deep transverse arrest T
284. During the third stage of labour, the placenta C. Non-engagement of head
should be expelled: D. Obstructed labour
A. By Brandth-Andrew method E. Oblique lie
B. Spontaneously
C. By controlled cord traction T
Page | 54
291. All are the signs of fetal compromise of CTG, 297. A primigravida wants evaluation for congenital
except: anomaly in her baby. The best time to screen
A. Persistent variable decelerations fetus for congenital anomalies in weeks by
B. Presence of accelerations T ultrasound is:
C. Loss of baseline variability A. 12-13
D. Fetal tachycardia (>160 bmp) B. 14-15
E. Fetal bradycardia (<100 bmp) C. 16-17
D. 18-19
292. A primigravida has undergone a ventous E. 20-22 T
delivery. On examination she was diagnosed to
have a third degree perineal tear. This includes 298. A 30-years primigravida is admitted at 38
injury to: weeks with severe preeclampsia. The CTG
A. Perineal skin only findings suggestive of severe fetal hypoxia is:
B. Perineal muscles + > 50% external sphincter A. Early decelerations
C. Perineal muscles + < 50% external sphincter B. Fetal heart rate variability of less than 08
D. Perineal muscles + external + internal beats/min
sphincter T C. Fetal tachycardia
E. Involvement of perineal muscles D. Late decelerations with minimal
variability T
293. All are the possible indications for classical E. Variable decelerations
caesarean section, except:
A. Postmortem caesarean section 299. A 26-years old multigravida at 36 weeks of
B. Large cervical fibroid T gestation has presented with vaginal bleeding.
C. Transverse lie On examination her uterus is tense and fetal
D. Fetal distress / prolapsed cord heart sounds are not audible. The diagnosis is
E. Placenta previa with large vessel on lower consistent with:
segment A. Incidental hemorrhage
B. Placenta previa
294. Infra-levator hematomas are the one in which C. Placental abruption T
blood collects in all spaces, except: D. Vasa previa
A. Involvement of vulva E. Vulval varicose veins
B. Extension of perineum
C. Paravaginal hematomas 300. Determinations of chorionicity in managing
D. Extension up to broad ligament T twin pregnancy is vital. This can most reliably
E. Extension into ischiorectal fossa done by ultrasound in:
A. Early 2nd trimester
295. Which one of the following viral infections B. Early 3rd trimester
carries highest maternal mortality: C. Early first trimester
A. Hepatitis A virus D. Late 2nd trimester
B. Hepatitis B virus E. Late first trimester T
C. Hepatitis C virus
D. Hepatitis D virus 301. The indication for immediate delivery to
E. Hepatitis E virus T prevent intrauterine fetal death in a Doppler
ultrasound pregnancy with severe IUGR is:
296. A 20-years old lady presented with the A. Reversed diastolic blood flow in umbilical
symptoms of fatigue, tearfulness and mood artery T
changes on 4th postnatal day. The symptoms B. Reduced amount of liquor
resolved spontaneously after 02 weeks. Her C. Reduced fetal limb movements
condition should be best labelled as: D. Reduced fetal respiratory movements
A. Anxiety E. Reduced S.D ratio on umbilical artery
B. Depression Doppler
C. Normal emotional changes
D. Postpartum blues T 302. The cardiac problem associated with very high
E. Puerperal psychosis perinatal mortality is:
A. Atrial septal defect
B. Eisenmenger’s syndrome
C. Mitral valve regurgitation
D. Mitral valve prolapse
E. Ventricular septal defect T
Page | 55
303. A baby is born to mother who is fond of A. Ductus arteriosus
keeping cats as pet animals at home. The baby B. Ductus venosus
has congenital anomalies in form of C. Foramen ovale
hydrocephaly, chorioretinitis and cerebral D. Umblical vein
calcification. The most likely maternal infection E. Umblical artery
is:
A. Herpes simplex virus 310. Insulin secretion in pregnancy is increased by:
B. HIV A. Progesterone
C. Parvovirus B. Estrogen
D. Rubella virus C. Growth hormone
E. Toxoplasmosis T D. Human chorionic gonadotropin
E. Prolactin T
304. The best management option for newborn baby
of hepatitis B +ve mother is: 311. Postpartum hemorrhage can be prevented by:
A. Both active & passive T A. Careful labour monitoring
B. Active immunization B. Augmentation of labour
C. Follow up to determine immunological status C. Active management of 3rd stage of labour
D. Investigation to find infective status D. Instrumental delivery
E. Passive immunization E. Provision of analgesia during labour

305. Select the drug of choice for prevention of 312. A collapsed grand multiparous admitted with
postpartum hemorrhage at a hospital setting: obstructed labour --------:
A. Methergin A. Fulminating sepsis
B. Mifepristone B. Severe preeclampsia
C. Misoprostol C. Ruptured uterus
D. Oxytocin T D. Antepartum hemorrhage
E. PG F2 alpha E. HELLP syndrome

306. A P1+0 attends OPD with history of exposure 313. Fetal tachycardia is because of:
to chickenpox in the infectious phase at 12 A. Maternal infection
weeks gestation. The best advice for her is: B. Fetal infection
A. Counsel about anomaly scan at 18-22 weeks C. Fetal hypoxia
B. Give varicella zoster IgG D. Ritodrine
C. Give acyclovir E. All of the above
D. Offer CVS (chorion villous sampling) or
amniocentesis for fetal infection 314. A baby is born with cataract and later develops
E. Test her for varicella zoster antibody T deafness. The most likely cause is:
A. CMV
307. A primigravida is booked at 20th week of B. Herpes simplex virus
gestation. Her Hb% is 6gm/dl. The peripheral C. Parvovirus
blood smear shows microcytosis, anisocytosis D. Rubella
and hypochromasia. The most important E. None of above
investigation to assess her iron stores is:
A. Serum iron 315. Which of the malarial parasite can cause severe
B. MCV fetal and maternal complications during
C. MCHC pregnancy:
D. Serum ferritin T A. Plasmodium vivax
E. TIBC B. Plasmodium vivax & malaria
C. Plasmodium malaria
308. The levator ani muscle: D. Plasmodium falciparum
A. Is attached to the pelvic brim E. Plasmodium ovale
B. Can act as sphincter of the urethra
C. Has no attachment to the perineal body 316. Delayed uterine involution can be caused by
D. Form the roof of ischioanal fossa any of the following expect:
E. Has motor innervation from the pelvic A. Full bladder
autonomic nerves B. Loaded rectum
C. C-section
309. At birth first change in fetal circulation D. Fibroid uterus
involves closure of: E. Retained products of conception
Page | 56
317. Acquired thrombophilia is most commonly B. External cardiotocography
associated with: C. Doppler ultrasound
A. Obesity D. Recording of fetal heart by scalp electrode
B. Smoking E. Fetal blood sampling
C. APS (anti-phospholipid syndrome)
D. Caesarean section 324. The best conservative surgical method for
E. Sepsis control of PPH in case of uterine atony is:
A. Caesarean hysterectomy
318. Vaginal breech delivery should not be B. Subtotal hysterectomy
attempted if: C. Brass suture
A. Fetal weight is 3500 gms D. Uterine packing
B. Fetal head is flexed E. Internal iliac artery ligation
C. Mild hydrocephalus
D. Fetal head is hyperextended 325. Forceps are contraindicated in:
E. Extended breech A. Preterm delivery
B. Brow presentation T
319. Which of the following vaginal infection is C. Face presentation
positively associated with preterm ----: D. After coming head delivery in breech
A. Bacterial vaginosis presentation
B. Trichomonas vaginalis E. Occipito-posterior position
C. Candida albicans
D. Herpes simplex infection 326. Which of the following is not associated with
E. Mixed bacterial infection bleeding from a low lying placenta:
A. Painless bleeding
320. What is the most important cause of dizygotic B. Uterus soft & non-tender
twin in current age: C. Vaginal bleeding with pain
A. Advance maternal age D. Presenting part will be free
B. High parity E. Bleeding can be recurrent in third trimester
C. Racial predisposition
D. Assisted reproductive technology 327. Placental abruption is not associated with:
E. Familial predisposition A. Hypertension
B. Smoking
321. A 22-years old primigravida 12 weeks pregnant C. Previous caesarean scar
presented with mild clinical ------ was 10g% D. Anticoagulant therapy
and red cell indices were as follow: MCV 66fl, E. Polyhydramnios
MCH 25Pg, MCHC 33g%. The type of anemia
she might have is: 328. Traumatic brain hemorrhage is most
A. Iron deficiency anemia commonly associated with:
B. Beta thalassemia A. Vacuum extraction at the pelvic outlet
C. Alpha thalassemia trait B. Outlet forceps
D. Hereditary spherocytosis C. Mid-cavity forceps deliveries
E. Sickle cell trait D. Neonatal coagulopathy
E. Spontaneous vertex delivery
322. A 38-years old woman with BMI 35 attended
antenatal clinic at 12 weeks gestation. She has 329. Epidural anesthesia is contraindicated in
past history of deep vein thrombosis. Select the patient with:
best treatment for her to -------- recurrence of -- A. Cardiac disease
--: B. Hydramnios
A. . C. Twin pregnancy
B. . D. Obstructed labour
C. . E. Ruptured uterus
D. .
E. . 330. Most commonest cause of postpartum
hemorrhage in grand multiparae is:
323. A primigravida admitted with ruptured A. Retained placenta
membrane, meconium stained liquor and 05 cm B. Rupture of uterus
cervical dilatation. Most appropriate fetal C. Atony of uterus
monitoring method for her: D. Inversion of uterus
A. Recording of fetal heart by pinards E. DIC
Page | 57
331. Perinatal deaths include: 339. The anterio-posterior diameter of inlet in
A. All stillbirths maternal pelvis is:
B. Babies dying within 28 days of birth A. 13.5 cm
C. All stillbirths & babies dying in 28 days of B. 12.0 cm
birth C. 11.0 cm T
D. All stillbirths & babies dying within 07 days D. 13 cm
of birth T E. 12.5 cm
E. All babies dying in one year of life
340. Folic acid is ideally given during:
332. The treatment of severe preeclampsia is: A. Peri-conceptional period
A. Magnesium sulphate T B. First trimester
B. Delivery of baby C. Second trimester
C. An antihypertensive drugs D. Third trimester
D. Renal dialysis E. Complete gestational period
E. Bed rest

333. The principle obstetric risk factor from mother


to child HIV transmission is:
A. Caesarean section
B. Vaginal delivery
C. Anemia
D. Advanced maternal age
E. High viral load

334. Platypeloid pelvis can result in:


A. Occipito-posterior position during labour
B. Obstructed labour
C. Occipito-transverse position
D. Deep transverse arrest T
E. Prolong labour

335. Umbilical cord prolapse is most likely with


which of the following:
A. Transverse lie T
B. Face presentation
C. Frank breach presentation
D. Complete breach presentation
E. Oblique lie

336. The partograph shows best:


A. Cervical dilatation
B. Presenting part
C. Status of membranes
D. Cervical dilatation & descend of fetal head T
E. Station of fetal head

337. Obstetric fistulae usually develops after:


A. Neglected obstructed labour
B. Caesarean section
C. Caesarean hysterectomy
D. Midline episiotomy T
E. Ventouse delivery

338. First trimester ultrasound is done for:


A. Dating the pregnancy
B. Fetal biometry
C. Knowing fetal anomalies T
D. Assessment of fetal wellbeing
E. Uterine anomalies
Page | 58

You might also like